You are on page 1of 130
Objecti Arithme 1 Introducing Online Shopping Now you can purchase/buy from our vast range of books and magazines onl © Publishers Publishers UPKAR PRAKASHAN (An 130.9001 : 2000 Company) 2/11A, Swadeshi Bima Nagar, AGRA-282 002 Phone : 2530966, 2531101, 3208693/94 Fax : (0562) 4031570 E-mail : publisher @upkar.in Website : www.upkar.in Branch Office 4840/24, Govind Lane, Ansari Road, Daryaganj, New Delhi-110 002 Phone : 23251844/66 © This book or any part thereof may not be reproduced in any form by Photographic, Mechanical, or any other method, for any use, without written permission from the Publishers. © The publishers have taken all possible precautions in publishing the book, yet if any mistake has crept in, the publishers shall not be responsible for the same. © Only the courts at Agra shall have the jurisdiction for any legal dispute, ISBN 81-7482-058-2~ ISBN 13 : 978-81-7482-058-7 Price : Rs. 190-00 (Rs. One Hundred Ninety Only) Code No. 447 PKAR PRAKASHAN (Printing Unit) Bye-pass, AGRA Contents 1. Numbers... nsiveinnie cer 3—25 2. HG and LGMG svisissreicsstiinsvecisnsiansncnesiaseoone 26—42 3. Fraction 4. Simplification. “5. Unitary Method. 6. Percentage... 7. Work and Time... . 124—144 8. Pipes and Cisterns. s, 145—157 9. Distance and Time & Questions on Train: . 158—182 10. Problems Based on Boats and Streams - 183—195 11. Ratio, Proportion and Proportional Division -. 196—219 12. Partnership. Zh .. 220—233 13. Square Root and Cube Root . 234—251 14. Average. +. 252—275 15. Profit and L . 276—310 16. Simple Interest ~ 311—337 17. Compound Interest -_ 338-—366 18. . 367—384 19. .. 385—414 20. Cuboid and Cube.. -. 415—430 21. Cylinder and Cone. 431—438 22. Sphere.......... 439—468 23. Stocks and Share: 469—485 24. Data Sufficiency... -. 486—502 25. Interpretation of Data. 503—S64 26. Statistics 565—587 27._To Find the Wrong Term in A Number of Series....... 588—604 28. Problems Based on Ages -. 605—624 Other Useful Books Upkar Arithmetic for Competitive Exams. Upkar Objective Arithmetic Review Upkar Instant Arithmetic Upkar Complete Arithmetic Upkar High Speed System of Basic Arithmetic Upkar Vedic Mathematics Sutra Upkar Mathematical Formulae (dee: ore gare wot od sare fie) ore attra qatar (wa afar ETA) Rs. 120/- Rs. 65/- Rs. 185/- Rs. 150/- Rs. 35/- Rs. 35/- Rs. 65/- Rs. 150/- Rs. 205/- Rs. 90/- Rs. 35/- Rs. 35/- Rs. 145/- Rs. 220/- Rs. 76/- OBJECTIVE MATHEMATICS 1 Numbers Introductory : 1 2. 3. 10. The numbers written by the symbols, 0, 1, 2, 3. 4, 5. 6, 7. 8 and 9 are known as digits. The numbers which are divisible by such as 2, 4, 6 etc, are known as even numbers. The numbers which are not divisible by 2 such as 1, 3,5, etc. are known as odd numbers. . The numbers 1, 2, 3, ........which are used for counting are known as the natural numbers. Thus 528 is a natural numbers but 0 - 3, 14.6, and 3/4 etc. are not natural numbers. . If zero is included in natural numbers the set so obtained is known as whole numbers. For example 0, 1, 2, 3, .....etc. are whole numbers but -1, —2, 3.24 are not whole numbers. . The whole numbers positive as well as negative are known as integers. For example -7, - 6, 5, .....0, 1, 2, 3, 4 etc. are integers butV5, 0.32, -1.2 and Fete. are not integers. . The numbers which can be expressed in the form of p/q, where p and q are integers but q # 0, are known as rational numbers. For example 25,-62, 2 2 z » 265° — 37 0 etc. rational numbers. . A number which can not be expressed in terminating decimal is known as an irrational numbers, For example Vs, 3+ V2, etc, are irrational numbers. The numbers which are either rational or irrational are known as real numbers. For example 3,-2, VTete. are real numbers. A natural number other than 1, is a prime number if and only if it is divisible by 1 and the number itself. For example 2 4s a prime number but 6 is not a prime furaber. 4A | O. Math 11. The natural number other than 1 is a composite number which is divisible by 1, the number itself and atleast one more other number. For example 6 is a composite number because it is divisible by 1, 2, 3 and 6. Note— 1. is neither prime number nor composite, 2. is only the number which is prime as well as even number 12. Dividend = (Divisor x Quotient) + Remainder. 13. In division performed by factors True remainder = First remainder + (First divisor x Second remainder) + (First divisor x Second divisor x Third remainder) + ........ For example on dividing 305 by 15, what will be the remainder. 3 | 307 5 [102-1 20-2 <. True remainder= | + (3 x2) =14+6 =7 14, The face value of a digit is known as the intrinsic value. For example the intrinsic value of 8 in 9872 is 8. 15. The value of a digit according to its particular place is known as place value. For example the place value of 8 in 9872 is 800. Tests of Divisibility : 1. A number is divisible by 2 if it ends in zero or in a digit which is a multiple of 2. For example 30, 36 etc. are divisible by 2. 2. A number is divisible by 3 if the sum of its digits is divisible by 3. For example sum of the digits of 402 is 6 and 6 is divisible by 3. Therefore, 402 is also divisible by 3. 3. A number is divisible by 4 if the last two digits i.e. tens and units are divisible by 4. For example last two digits of 372 are 72 and 72 is divisible by 4. Therefore, 372 is also divisible by 4. 4. A number is divisible by 5 if it ends in zero or 5. For Sxample 205, 320 etc. are divisible by 5. 5. A number is divisible by 6 if it is divisible by 2 as well as by 3. For example 510 is divisible by 2 as well as by 3. Therefore, 510 is divisible by 6. 6. To check whether the given number is divisible by 7, the unit digit of the given number is doubled and then it is subtracted from the number obtained after omitting the unit digit. If the remainder is divisible by 7, then the given number is also divisible by 7. O. Math | SA If the given number is large enough, then the process is repeated till such a small number is obtained which can be easily checked whether it is divisible by 7. Example 1. 343 is to be checked whether it is divisible by 7. Solution : On doubling the unit digit 3 of 343, we get 6 then 34-6 =28 Since 28 is divisible by 7, hence, 343 is divisible by 7. Example 2. 5499838 is to be checked whether it is divisible by 7. The unit digit of the given number is 8. Solution : On doubling 8 and subtracting from the number which is obtained after omitting the unit digit, we get 549983 —16 = 549967 54996-2x7 = 54982 5498-22 = 5494 549-2x4 = 541 54-2x1 = 52 Since 52 is not divisible by 7, therefore, 5499838 is also not divisible by 7. 7. Anumber is divisible by 8 if its last three digits i.e., hundreds, tens and units, are divisible. by 8. For example the last three digits of 5248 are 248 and 248 is divisible 8, hence 5248 is also divisible by 8. 8. A number is divisible by 9 if the sum of its digits is divisible by 9. For example the sum of the digits of 576 is 18 and 18 is divisible by 9. Therefore, 576 is also divisible by 9. 9. A number is divisible 10 if it ends in zero. For example the last digit of 410 is zero, therefore, 410 will be divisible by 10. 10. A number is divisible by 11 if, the difference between the sums of the digits in the even and odd places is zero or a multiple of 11. For example in 1573, the sum of the digits in odd places is 5 +3 ie. 8 and the sum of the digits in even places is 7+ 1 i.e. 8. Therefore, the difference between there sums is zero. Hence 1573s divisible by 11. Similarly in 9174, the sum of the digits in odd places is 4+ 1 ie. 5 and the sum of digits in even places is 7 + 9 i.e. 16. Now the difference between there sums in 16 - 5 i.e. 11 which is a multiple of 11. Therefore, 9174 is divisible by 11. This One ion 6A | O. Math Recurring Decimal : If in a decimal number a digit or a set of digits is repeated again and again, the decimal is known as recurring decimal. In short it is expressed by putting dots over the first and last digits which are repeated. For example. 2.666... .728372837283... and 7.342424... = 7.342 Addition and Subtraction of Decimal : While doing a sum of addition or subtraction of decimals, the following steps should be executed : 1. The numbers should be placed in columns i.e. ones under ones, tenths under tenths etc. 2. The decimal points should form one column. 3, The numbers are added or subtracted in the same way as whole numbers. 4, The decimal point is put in the answer directly under the column of decimals. Muitiplication of Decimals : Ignore the decimal points and multiply the two numbers as the whole numbers . In the product, put a decimal point after as many places from the right as there are in the multiplier and multiplicand together. Division of Decimals : 1, When the divisor is a whole number, perform the division as in the case of whole numbers. In the quotient put a decimal point after as many places from the right as there are in the dividend. 2. When the divisor is a decimal, move the decimal points of the divisor to the right until it becomes a whole number. Next move the decimal point of the dividend the same number of places to the right adding zeros if necessary. Then proceed to divide as given above. Use of Some Formulae in Simplification : a? —b2 =(a+ b) (a—-b) a? +2ab+ b? =(a+ by a? -2 ab+b? =(a-by a3 + b3 =(a+ b) (a? - ab + b*) a —b3 =(a—b) (a? + ab +b?) @+h4 3-3 abe=(at+b+c) (a +b + 2 -ab -be -ca) Example 1. What is the unit digit in the product of 237 x 72 x 28x 15? AWN O. Math | 7A Solution : The’ unit digit of the numbers 237, 72, 28 and 15 are 7, 2; 8 and 5 respectively. The product of 7, 2, 8 and 5 = 560. Be The unit digit in the required product is 0. Ans. Example 2. If a number is divided by 84 the remainder is 37. What will be the remainder if it is divided by 21? Solution : Since 84 is divisible by 21, therefore, (the given number-37) will also be perfectly divisible by 21. So it is the remainder i.e. 37 which is to be considered when divided by 21. But we know if 37 is divided by 21, the remainder-is 16. Hence when the given number is divided by 21, the remainder will be 16. Ans. By Short-cut Method : Number =84K+37 1x4K + (21 + 16) =21(44K+1)+16 ~. Remainder when the number is divided by 21 is 16. Example 3. What is the sum of all odd numbers between 1 to 50? Solution : By Short-cut Method : The odd numbers between 1 to 50 are— 1, 3, 5, 7, 9, 11, 13, 15, 17, 19, 21, 23, 25, 27, 29, 31, 33, 35, 37, 39, 41, 43, 45, 47 and 49. Thus the number of odd numbers = 25 -. Sum of all these odd numbers = (25)? = 625 Ans, Note— 1. If we want to find the number of odd numbers between two numbers whose last number is even, then last number is halved and result so obtained is number of odd numbers. For example the number of all odd numbers from 0 to 50 is 25 because half of last number 50 is 25. 2. If the last number is odd, then in order to find the number of odd number, 1 is added to the last number and then it halved. For example the number of odd numbers from 0 to 25 is x (25 +1) ie. 13. 3. The sum of odd numbers = (number of odd numbers)? 8A | O. Math Example 4. What is the sum of all even number between | to 40 7 Solution : The even numbers from 1 to 40 are 2, 4, 6, 8, 10, 12, 14, 16, 18, 20, 22, 24, 26, 28, 30, 32, 34, 36, 38 and 40. No. of these even numbers = 20 Sum of even numbers = 20 x 21 = 420 Ans. Note— 1. In order to find the number of even numbers, last even number is halved. For example in the question given above the last even number is 40. Hence the number of even numbers from | to 40 is 20. 2. Sum of all even numbers = n (# + 1) where n is the number of even numbers. Example 5. Which digits can be the last digit of an exact square number ? Solution : +: @ =0, 1? =1, 2? =4, 3? =9, 4? = 16, 5? = 25, 6? = 36, 7? =49, 8 = 64 and 9? = 81. The last digit of a square number may be, 0, 1, 4, 5, 6, and 9 only Ans. Example 6. How many numbers between 1 and 400 are such which are exactly divisible by 7 7 Solution : If 400 is divided by 7, the quotient will be 57 and remainder 1. Hence between I and 400, there are 57 numbers which are exactly divisible by 7. Ans. Example 7. What is the least number of 4 digits which is exactly divisible by 3 ? Solution : The least number of 4 digits = 1000 But 1000 is not exactly divisible by 3 Therefore, we should consider about the number 1001, 1002, ....etc. for the least number of 4 digits. We see that 1002 is exactly divisible by 3. Hence the least number of 4 digits which is divisible by 3 is 1002. Ans, Example 8. What is the difference in intrinsic value and local value of 5 in 7502 7 O. Math | 9A Solution : The local value of 5 in 7502 = 500 and intrinsic value of 5 in 7502 =5 * Required difference = 500-5 =495 Ans. Example 9. What is the least prime number of three digits ? Solution : Least number of 3 digits = 100 But 100 is not a prime number Now the number next to 100 is 101 and it is a prime number. Therefore, the least prime number of 3 digits is 101. Ans, Example 10. What is the sum of all the natural numbers from 1 to 40 ? Solution : By Short-cut Method : Sum of all the natural number from 1 to 40 _ 40x41 2 = 820 Note—Sum of all the natural numbers from 1 to n= ae) Example 11. If a number is successively divided by 5 and 7, then the remainders are 3 and 4 respectively. What will be the remainder if the same number is divided by 35? Solution : True remainder = First remainder + (First divisor x Second remainder) Ans. Example 12. If (4137)7> is simplified, what will be the last digit in the answer ? Solution : The last digit in (4137)’3 will be same as the last digit in (753. But we know (7)! = 7, (7)? = 49, (7)3 = 343, (7)* = 2401, (7) = 16807 Hence last digit is repeated after each 4 index 10A | O. Math The last digit in (7)’53_ = the last digit in (7)’5%*! = the last digit in (7)! =7 Ans. By Short-cut Method : In such questions if index (power) is n, then after dividing (n — 1) by 4, remainder is calculated then by the following formula the unit digit is found. Unit digit in (4737)"53 = (unit digit of 4737) "minder +1 Here, n = 753, therefore, n-1 = 752 After dividing 752 by 4, the remainder = 0 Unit digit of (4737)78 = (7)°+! = 7. Ans. Example 13. What is the sum of first four prime numbers ? Solution : Sum of first four prime numbers 2434547 17 Ans. wu Example 14, A number of 6 digits is formed by repeating a two digits number three times (Such as 565656 or 121212). By which number, such number is always divisible ? Solution : Let the two digits number be 10x+ y ~. Number of 6 digits in the form as stated above = (10x +y) x 10000 + (10x+ y) x 100 + (10x+ y) (1Ox + y) [10000 + 100 + 1] = (10x +y)x 10101 ~. Such number is always divisible by 10101 Ans, Example 15. What is the number of prime factors of (6)! x (7)!7 x (55)??? Solution : (6)! x (7)"7 x (55)" No. of prime factors " 210 x 310 x TIT x 527 x 1127 10+ 10+ 17+27+27 91 Ans. uu O. Math | 11A Example 16. On converting the binary number 1101101 into decimal form what will we get ? Solution = (1101101). = 1x2 41 x254+0x 2441x341 x 2240x2!+1 x20 = 6443240484+4+0+1 = 109 Ans, Formula—The sum of both digits of a two digit numbers is S. If the digits of the number are interchanged the number so found is greater than the original number by p, what is the orginal number ? Proof —Let the number be 10x+ y ne xty=S i) and (10y +x) (10x +y) = p He Oy-9x = p a2 a, ms YX EG +i) From equations (i) and (ii) x= SP) “XB and ye Ste Original number = 0 S# +%te _ US-p - 2 Example 17. The sum of both digits of a two digit number is 7. If the digits of the number are interchanged, the number so formed is greater than the original number by 27. Find the original number. Solution : Let the number be 10.x+ y. xty=7 A) and (10y +x)-(10x+y) = 27 or, yox =3 sii) :. From equation (i) and (ii), x = 2andy=5 Original number = 25 Ans, 12A | O. Math By Short-cut Method : Original number = us=e Here, S =7 and p = 27] x 7=27 = 2 = 25 Example 18. If (a" + b*) is divisible by (a + &), then what will be the value of n, positive or negative ? Solution : If n is odd, then (a" + b”) will be divisible by (a + 5). For example, if n= 3, then a? + b3 =(a+ b) (a? —ab + b?) 4 «. (a3 + b3) is divisible by (a +b) and if n= 5, then a’ + bS =(a+ b) (a4 — ab + ab — ab? + b4) «. (a5 + BS) is divisible by (a + 5). Thus, for any odd value of n, (a” + b") is divisible by (a+ 5). Ans. O. Math | 13A Exercise Directions—In each of the following questions a number of possible answers are given out of which one is correct. Find out the correct answer. 1. What is the unit digit in the product of 207 x 781 x 39 x 94? (A) 9 @) 1 C7 @) 2 © 4 2. If the product of 305, 211, 702 and 13 is calculated, then what will be the unit digit in the product ? (A) 3 (B) 0 © @) 2 ©) 3 3. If 962 is multiplied by 307 and the product so obtained is multiplied by 103 then what will be the unit digit in the last product ? (A)-2 @B) 3 7 @) 0 © 8 4. A number when divided by 296 gives a remainder of 75. What will be the remainder when the same number is divided by 37 ? (A) 3 @B) 5 (1 @) 6 © 2 5. A number when divided by 899 gives a remainder of 63. If the same number is divided by 29, then the remainder will be : (A) 2 (B) 5 © 13 @) 28 (E) None of these (Asstt. Grade Exam. 2006) 6. If a number is exactly divisible by 85, then what will be the remainder when the same number is divided by 17 ? (A) 3 (B) 2 (4 @) 0 ® 5 7. What will be sum of all odd numbers between 1 and 32 ? (A) 256 (B) 128 (C) 25 @) 16 ® 512 8. Avcouple has six married sons each of which has 4 children. How many members are there in that family ? (A) 32 () 35 (C) 43 @M) 42 © 38 (R.R.B, 2004) 9. What will be the sum of all odd numbers between 30 and 50 ? (A) 625 (B) 400 (C) 225 @) 500 © 540 14A | O. Math 10. 13, 14, 16. 17. 18. 19. 20. What will be the sum of all even numbers from | to 20? (A) 120 (B) 100 (© 110 @) 115 ©) 128 . What will be the sum of first 25 even numbers ? (A) 156 (B) 204 (C) 308 (D) 448 ©) 650 . What will be the sum of all the even numbers between 1 and 60 ? (A) 870 (B) 960 (C) 840 @) 720 () 852 What will be the sum of all even numbers from 41 to 80 ? (A) 640 (B) 2080 (©) 1220 @) 1640 ©) 420 Which digit can not be on the piace of unit digit when any number is squared ? (A) 0 @) 4 ©1 @) 9 ® 2 . Which of the following digits can be on the place of unit digit when any number is squared ? (A) 7 (B) 8 (C) 3 @) 5 © 2 If a number is cubed, then which digit can be on the place of unit digit ? (A) 5 @)7 (C) any digit from 0 to 9 @) 4 © 3 How many digits are required to write the numbers from 1 to 100 ? (A) 169 (B) 192 (©) 99 @) 198 (E) None of these How many numbers between | and 100 are such that they are divisible by7? (A) 14 @®) 12 © @) 3 (©) None of these (R.R.B. 2007) How many numbers between 200 and 300 are such that they are divisible by 13 ? (A) 7 @®) 10 ©8 @) 6 ® 5 How many numbers upto three digits are divisible by 19 ? (A) 48 (B) 47 (C) 25 @) 31 ©) 52 21. 22. 23. 24. 27. 28. 29. 31. O. Math 115A Find the smallest number of four digits which is divisible by 13 ? (A) 1014 (B) 9984 (C) 988 ©) 1001 @® 1027 Find the largest number of five digits which is divisible by 17 ? (A) 99999 (B) 99960 (C) 99994 @) 10013 ) 10047 What is the smallest number if that is subtracted from 342, the remainder is exactly divisible by 8 7 (A) 4 @B) 2 (7 ©) 1 © 6 What should be subtracted from 32575 so that the remainder is exactly divisible by 9? (A) 4 (B) 2 © @) 3 () None of these (P.O. 2007) .. What is the difference between the local value and intrinsic value of 9 in 3901? (A) 900 (B) 9 (C) 891 @) 40 ©) None of these . What is the difference between the local value and intrinsic value of 7 in 4375? (A) 7 (B) 10 (©) 100 @) 63 ©) None of these What is the largest prime number of two digits ? (A) 93 (B) 97 (© 89 @) 101 @) None of these ‘What is the largest prime number by which 871 is exactly divisible ? (A) 13 @B)1 (©) 67 @) 871 ©) None of these Which one of the following is a prime number ? (A) 15 @®) 33 © 21 @) 9 ® 3 ). How many prime numbers are there between 70 and 100 ? (A) 7 @B) 8 ©5 @) 6 ® 3 The total number of prime numbers between 0 and 100 is (A) 31 (@) 29 (© 25 @) 23 ©) None of these (Air Force Technical Trade, 2005) 16A | O. Math 32. 33. 35. 36. 37. 38. 38. 4. Which one of the following is the pair of twinprimes ? (A) 5,11 @) 7,1 © 1,17 @) 1,3 ©) 17,19 Which one of the following is the pair of co-primes ? (A) 14,35 (B) 24,32 (C) 20, 25 ©) 10,21 €) 30,36 . What is the sum of all integers from | to 20 ? (A) 220 (B) 190 (© 210 @) 105 ® 110 What i$ the sum of all integers from 30 to 50? (A) 870 (B) 840 © 830 ©) 820 © 810 The difference between the greatest and the least numbers of eight digits which begin with 8 and end with 6 is— (A) 99999999 (B) 10000000 (C) 80000006 (D) 999999 (E) None of these The difference between the greatest and the least numbers of 5 digits 0, 2, 3, 6 and 7 is— (A) 50953 (B) 35905 (C) 55953 @) 95821 ©) 39508 It is given that 23 + 1 is exactly divisible by a certain number. Which one of the following is also divisible by the same number ? (A) 2%41 (B) 2'°-1 (C) 21641 @) 7.2% () None of these S.S.C. Clerk Grade Exam. 2006) The square of an odd number is— (A) Always an even number (B) Always an irrational number (C) Always prime number (@) Some times even and some times odd (E) Always an odd number ). Which is the greatest out of the following numbers ? (A) (2+242)? ®) ((2+2)?7 (C) @x2x2)? @) 4 ® x* (92)? — (85)? is divisible by— (A) 17 @) 11 ©) 10 @) 13 ©) 59 42. 43. 45. 47. 49. O. Math | 17A The digit in the blank space of the number 34 * 7 so that the number is divisible by 11, will be— A 3 @) 6 (7 ©) 8 ® 1 Anil was asked to divide a number by 7 but he multiplied the number by 7. If the result he got, was 7, the actual answer should be— (a) 1 @) 47 os ©) 7 ® 4 . Ifa *b =a? + b?, then -3 * 5 is equal to— (A) 16 ) 34 ©8 © 15 ® 12 If the sum of a number of two,digits and a number formed by reversing the digits is 99, then what is the sum of the digits of the original number? (A) 9 () 81 (©) cannot be known @M) 18 ©) None of these (P.O. Exam. 2008) .. Out of 500 people, 300 know only Hindi and 120 people know only English. How many péople know Hindi and English both ? (A) 70 (B) 100 (©) 80 ©) 60 (©) None of these In the product of 459 x 4 * x 787 x 483 if the digit in the unit place is 7, then what will come in place of the asterisk ? (A) 3 @)7 ©9 @) 5 ©) None of these (A.A.O. Exam. 2007) . A number being successively divided by 9, 11 and 13 leaves 8, 9 and 8 as remainders respectively. If the order of divisors is reversed then remainders will be— (A) 8,9,8 B) 9, 8,8 © 10, 1,6 @) 10,8,9 ®) .None of these (S.S.C. Asstt. Grade Exam. 2005) If the sum of the digits of any number, lying between 100 and 1000, is subtracted from the number, the difference is always divisible by— (A) 12 @)9 C6 @) 2 (E) None of these (S.S.C. Asstt. Grade Exam. 2007) 18A | O. Math 50. 51. 52 53. 54. 35. 57. 58. What is the value of k in the number 5k 3457, when the number is completely, divisible by 11 ? (A) 1 (B) 2 ©3 @) 5 ©) None of these (S.S.C. Asstt. Grade Exam. 2006). A 4-digit number is formed by repeating a 2-digit number such as 2525, 3232, etc. Any number of this form is always divisible by— (A) 11 (@)7 © 13 @) 101 () None of these (S.S.C. Asstt. Grade Exam. 2004) A number, when successively divided by 3 and 5, leaves remainders of 2 and 1. When the same number is divided by 15, the remainder is— (A) 1 () 2 ©s5 @)7 (E) None of these (S.S.C. Clerk Grade Exam. 2005) The numbers 1, 3, 5, 25 are multiplied together. The number of zeros at the right end of the product is— (A) 1 (@) 0 (C2 @) 3 © None of these (S.S.C. Clerk Grade Exam. 2006) In orde: to make the statement (5 %) x (%% = 12 true, the missing terms in place of * and ® are respectively— (A) 9and3 (B) 12and7 (©) 10and2 @) 9and2 () None of these S.S.C. Clerk Grade Exam. 2003) If 30 x 0103 is divisible by 11, then the valute of x is— A) 1 (B) 4 6 ©) 7 €) None of these (S.S.C. Clerk Grade Exam. 2004) What is the unit number in the value of (729)? ? (A) 9 @®) 8 ©7 ©) 6 ©) None of these What will came in place of unit digit in the value of (7)35x (3)7! (11) 35? (A) 0 @®) 3 @©1 @) 6 (E) None of these What is the unit number in (3)®? + (2)®7 ? (A) 0 @) 5 (C) 2 @) 3 ©) None of these O. Math | 19A 59. Sum of the digits of a two digit number is 12. If the digits of the number are interchanged then the new number is 36 less than the original number. What is the original number ? (A) 95 (B) 72 (C) 82 @) 84 (©) None of these 60. 7!2_ 4!2is exactly divisible by which of the following ? (A) 36 @) 35 (C) 34 @) 33 (E) None of these 61. For which of the following values of n, the number a” + b” is exactly divisible by (a +b)? (A) 732 (B) 416 (© 709 () 1208 (E) None of these Answers 1L@) 2@) 3(4 4© 5@) 6@) 7A) &8@® 9.(B) 10.(C) 11.) 12(A) 13. 14.) 15.) 16) 17.(B) 18(A) 19.) 20.@) 21.0) 22.© 23.) 24(A) 25.(C) 26.(D) 27.(B) 28.(C) 29.(B) 30.(D) 31.(C) 32.6) 33.(D) 34.(C) 35.(B) 36.(D) 37.(C) 38.(A) 39.(B) 40.(B) 41.(E) 42.0) 43.(C) 44.(B) 45.(A) 46.(C) 47.(A) 48.0) 49.(B) 50.(B) 51.(D) 52.(C) 53.(B) 54.(D) 55.(B) 56.(A) 57.(C) 58.(B) 59.(D) 60.(D) 61.(C) Hints 5. Number = 89%K + 63 = 31x29xK+29 x24+5 = 2931K+2)4+5 «. The remainder when the number is divided by 29 is =5 Ans. oR The number of odd numbers from 1 to 50 = 25 os Sum.of odd numbers from 1 to 50 = (25)? = 625 and the number of odd numbers from 1 to30 = 15 4 Sum of odd numbers from 1 to30 = (15)? = 225 Sum of odd numbers from 30 to 50 = 625-225 = 400 Ans. 12. + Last even number of the even numbers between 1 and 60 is 58 Number of even numbers between 1 and60 = 29 Sum of even numbers between 1t060 = 29 x30 870 Ans. 20A | O. Math 13. Number of even numbers from 1 to 80 = 40 Sum of even numbers from | to 80 = 40x41 1640 and number of even numbers from | to40 = 20 .. Sum of even numbers from | to 40 20 x21 = 420 Sum of even numbers from 41 to 80 = 1640-420 = 1220 Ans. 14. If a number is squared, it will end with either 0 or 1 or 4 or 5 or 6 or 9. Hence it will never end with 2. Ans, 16. -:03=0, 13 =1, 23 =8, 33 =27, 43 =64, 53 = 125, 63 =216, P = 343, 8? = 512 and 9 =729 -. If a number is cubed, any digit from 0 to 9 may be in place of unit digit. Ans. 17. Digits required to write from 1 to9 = 9 Digits required to write from 10to99 = 90 x2 = 180 and Digits required to write 100 = 3 -. Digist required to write from 1 to 100 = 9418043 = 192 Ans. 19, The number of the numbers between 1 and 200 and divisible by 13 = 15 and number of the numbers between 1 and 300 and divisible by 13 = 23 “. Number of the numbers between 200 and 300 and divisible by 13 =23-15=8 Ans, 20. There is number of | digit. which is divisible by 19 There are S numbers of only 2 digits which‘are divisible by 19 There are 52-5, i.e. 47 numbers of only 3 digits which are divisible by 19 +. Total numbers of those numbers which are divisible by 19 and of upto three digits = 5 + 47 = 52 Ans. 21. The smallest number of 4 digits = 1000 If 1000 is divided by 13, the quotient is 76 and the remainder is 12. But on considering the numbers greater than 1000, ie. on 1001, 1002, etc. it is found that 1001 is exactly divisible by 13. <. The required number is 1001. Ans, 22. The largest number of five digits = 99999 If 99999 is divided by 17, the quotient is 5882 and the remainder is 5. Hence the greatest number of 5 digits, which is divisible by 7 = 99999 — 5 i.e. 99994, Ans, O. Math | 21A 23. If 342 is divided by 8, the remainder is 6. Hence the required number = 6 Ans. 24. The sum of digits of 32575 = 3+2+5+7+5 = 22 Since a number is divisible by 9 if the sum of its digits is divisible by 9. If 22 is divided by 9 the remainder is 4. There if 4 is subtracted from the given number the remainder will be exactly divisible by 9. Ans. 27. The largest number of 2 digits = 99 But 99 is not a prime number The numbers smaller than 99, are 98, 97, ......etc. Out of these number 97 is the prime number. Ans. 28. 871 = 13 x 67 Out of these numbers, 67 is the largest prime number. Ans. 30. Following are the prime numbers between 70 and 100- 71, 73, 79, 83, 89 and 97. Ans. 31. The following numbers are the prime numbers between 0 and 100. 2, 3,5, 7, 11, 13, 17, 19, 23, 29, 31, 37, 41, 43, 47, 53, 59, 61, 67, 71, 73, 79, 83, 89 and 97. Ans. 32. Twinprimes are the pairs of those prime numbers whose difference is 2. In the given alternatives, (17, 19) is such a pair of two prime numbers whose difference is 2. Ans. 33. Co-prime is the pair of such numbers. whose common factor is nothing except 1. Out of the given alternatives only (10, 21) is such a pair in which common factor is nothing except 1. Ans. 35. Sum of all integers from 1 to 50 = a = 1275 and sum of all integers from 1t0 29 = 2230 = 435 Sum of all integers from 30 to 50 = 1275-435 = 840 Ans. 36. The greatest number of 8 digits which begins with 8 and ends with 6 = 89999996 and the smallest number of 8 digits which begins with 8 and ends with 6 = 80000006 Required difference = 89999996 — 80000006 = 9999990 Ans. 22A | O. Math 37. The greatest number with the digits 0,2,3,6,7 = 76320 and least number with the digits 0, 2,3,6,7 = 20367 z Required difference = 76320-20367 = 55953 Ans, Note—When 0 is also among the given digits, then to write the smallest number then zero is written after the next number bigger number. 38. a} +b) = (a+b) (a? -ab+ b?) * Qe) = (23241) (24-22 x1 41) .. 295+ 1 is also divisible by the same number Ans, 40. (2+2+2)? = (0)? =36 [2 +2)2P = (42)? =(16)? = 256 (2 x2x 2)? = (8)? =64 ()3 = 64 and @)4 = 81 «. [2 +2)°} is the largest number Ans, 41. (92)? — (85)? = (92 +85) (92 - 85) = 177x7 3x59x7 Ans. 42. If the given number is divisible by 11, then the sum of 7 and 4 must be same as the sum of 3 and the missing number. Therefore, there should be 8 in place of missing number. Ans, 43. If the product of a number and 7 is 7, then the number is 1. Hence on dividing 1 by 7, the result will be }. Ans, 44, -3*5 = (32452 = 9+25 = 34 Ans. 45. Let the number of two digits is 10x+ y The number formed by reversing the digits = 10y+x l0x+ y+ l0y +x = 99 or, lixt+ ly = 99 x+y = 2 =9 Ans, 46. Total number of people = 500 No. of people who know only Hindi = 300 and No. of people who know only English = 120 47. 48. 49. 50. 51. 52: 53. 55. O. Math | 23A «. No. of people who know Hindi and English both = 500- (300 + 120) = 80 Ans. The unit digits of 459, 787 and 483 are 9, 7 and 3 respectively. and 9 x 7 x 3 = 189 in which unit digit is 9 Since the unit digit in the whole product is 7 Therefore, in place of asterisk there should be 3 because 9 x 3 = 27 Ans, True remainder = 8+(9x 9)+(8X 9X11) = 8+81+792 = 881 13 | 881 11 | 67-10 9 | 6-1 0-6 New remainders are 10, 1 and 6 Ans. Any number between 100 and 1000, may be written as 100m + 10n +k where 0< m t z | I) z | or, 50A | O. Math 15. Which one of the following groups is in descending order ? 135 153 313 331 M522 @z55 ©2455 OF.54 315 ®© 5°3°7 16. Which one of the following is in ascending order ? U1 16 1 16 Hu 16 A igisn = ®) 9° q4°24 © 31-14" 19 16 uu u 16 11 ®) 19° 21° 14 ® 21° 19° 14 17. $0F 70s tess than $ of 112 by— (A) 42 (B) 24 (2 @) 40 (©) None of these 18. Which one of the following fractions is greater than 35 35 13 aL 19 A 7 ®B) x © tor ©) 4 9 ® 19, What decimal fraction of 0.3 metre is equal to 6 cm ? (A) 0.18 @) 0.12 (©) 02 @) 0.02 () None of these 20. Which of the following fractions is the smallest ? Sl, i121 3L a A) 356 ®) 512 © tg ©) 9 17 ©) 6 21. Which’of the following fractions is the smallest ? (A) 27 B) ol © a 2 p12, < 1535 170 ©) 56 18 * ® 95 22.. Which one of the following fractions is greater than : ? 27 20 16 33 A) 99 ®) 61 © 45 ©) t00 51 ® isa 54A | O. Math 25.(C) 26.(B) 27.(A) 28.(C) 29.(€) 30.(A) 31.(C) 32.(C) 33.(D) 34.(C) 35.(B) 36.(A) 37.(A) 38.(A) 39.(C) 40.(CE 41.(C) Hints 1. LCM. of3,5, 11 and 17 = 2805 2 _ 2x935_ 1870 3 = 3x935~ 2805 3 _ 3x561_ 1683 5 ~ 5x561~ 2805 & _ 8x255 _ 2040 11 ~ 11. x2557 2805 M1 _ 1x 165_ 1715 and 17 ~ 17x 165> 2808 Since the denominators in all these fractions are equal but among numerators 2040 is the largest. The largest fraction = * Ans, 2 LCM. of7, 8, 16 and35 = 560 5 _ 5x70 _ 350 8 ~ 8x70 560 21 _ 21x16 _ 336 35°" 35x16 560 9 _ 9x35. 315 16 ~ 16x35 560 6 _ 6x80_ 480 and 7 = 7x80" 560 «. Difference between the largest and the smallest fractions 6 9 ~ 716 3. = 112 Ans, 4, Let the number be 1 1 1 sof! 5 1 4 ve 1-5 =5 Number = 20 +4 = 0x3 225 Ans. 58A | O. Math In all these fractions the numerators are same and the denominator 1536 is the largest. Therefore, the smallest fraction is > at 312 Ans, me 16 15 1 22. *. 45 7 45% 3 7.701 me 82 < 81 3 2 2001 61 < 60% 3 33. 33 1 100 < 99 % 3 SSL and 154 ~ 153% 3 Ame. 11 L 33. 576 30 Capacity of the tank = 8 +55 240 litres Ans. 24. Weight of empty bucket + weight of full water = 17kg and weight of empty bucket + weight of half water = 13-5kg Weight of half water = 17-135 3.5 kg Weight of empty bucket = 13-5-3-5 = 10kg Ans, 25. Small fraction = bigger fraction - difference 5 Tv = 1645-125) 1 =4 8 Ans. 1 26. Colourless part of the pole = 1— (i+4 stat +t) = 2B ~ 60 Since the length of the colourless part = 65cm Length of the whole pole = 65 +2 ao = 65x B 8 8 | implification ‘The word simplification refers to convert long fractions or expressions in one fraction or number. For solving the questions on simplification the golden rule to remember is UBODMAS. Each letter of the word UBODMAS stands as follows : 1, U for underline i.e., the underlined portion of the given fraction should first be solved. 2. B for brackets i.e., after solving the underlined portion, the brackets should be removed. Different types of brackets should be removed. in the following order : First solve « ) then solve { }sand lastly [J 3. O for of. 4. D for division; it should be converted into multiplication by reversing the term on the right side of the sign. 5. M for multiplication. Before solving multiplication, division should be solved. 6. A for addition. 7. S$ for subtraction. The answer should be reduced to the lowest term of fraction. Example 1. Simplify : 73 - [53- { ae (332) }] Solution : 66A | O. Math (5-2 x PHA? (5-2-3 x P+ sy 52 (53 x Py? (55x 78y2 (When a number is brought to numerator from denominator its sign of index is reversed) = S3XT x TOX TD 9g 5-5 K-52 y¢ BX -S2 SZ x PA y 5252 x 7-20 522+ 252 721-20 53 xT 25 x7 175 Ans. ounud Example 7. What is the lowest value of (-DEDEMDEDED! “HCD EDODODOD ¢ oC 3) (ie) [because (a + b) (a-b) = a2 —b?] 3,8, = 4%9% i =3 Ans. Example 8. i I L 1 aS What is the value of (1 +) (: +) (: -4) . (: a) 2 Solution : L L i (1-7) (0-3) G3) C5) w Syty8y 18 FMR MQ rr 19 6 = 19 Ans, Example 9. “3 What is the value of (-a) on simplification ? Solution: r 3 (- oe) = (-343% (to convert negative index into positive = (3 the fraction is inverted) = C7? 49 , Ans. 70A | O. Math 17, 19. 20. 21. 22. 23. The value of 16 [42 (35 -(17- 4-7) -5} - 2] is— (A) 384 (B) 480 (C) 400 (D) 304 (E) None of these The value of (7482 +¢252)3 +3 x -748 x 2521. value oF (748? +2. 748 x 252 + (252)? (A) 5 (B) 03 (©) 102 ()! ©) None of these 1.073 x 1-073 ~ 927 x 927 347! x me fea 1-073 — 927 27)" x(3y 1 1 A A) 2 ®) 5 © 25 ©) 35 (E) None of these 13 X13 13K 7413 x 7-7X7 5 The value of The value of 13 x7 2 2 10 19 (A) "pr (B) 7 (C) 85 0 @) "Oy 29 ® 19] The value of 225 x 0-12 — 0-04 x 225 is— (A) 1-80 (B) 27 (Cc) 9 @) 18 () None of these The value of (43)4 + (42) is— (A) @)" ®) @)” © Ss @) (4)'6 (©) None of these 84 + 36) x(8 +6) . The value fay x11) -(19 x3) 8 (A) 15 (B) 90 © 21 @) 1625 @® 2625 (25 x8)—(16 x5) The value of 124556 is. (A) 20 (B) 180 (©) 250 @) 30 ©) None of these 18-3 x4+2. . The value of 6x5-3x8! 3 2 4 31 Ay B)3 O35 @) 338 25 © 33 T4A | O. Math S 1 joo nD 2{8|5| oia| 1,9 12 “40 333_ 513. = 16072160 Ans. 1 538 x -538 - -462 x -462 _ (-538)? - (-462)2 . 1--924 ~ 1--924 (538 + -462) (-538 — -462) 1--924 because a? — b? =(a+ b) (a—b) _ 1x06 ~ 016 =1 Ans. 4-669 X 4-669 — 2-331 x 2:331 (4069)? + (2-331)? + 4-609 x 4-662 (4-669)? — (2-331)? (4-669)? + (2331)? +2 x 4.669 x 2331 _ (4669 + 2-331) (4-669 ~2-331 (4-669 + 2-331)? because a? —b? =(a+ b) (a—b) and a? + b? + 2ab=(a+b)? _ 4:669 -2:331 ~ 4-669 +2-331 2338 = 7.900 = 9334 Ans. 7-84.x 7-84 — 2-16 x 2-16 2 784-216 *°> _ (7-84)? ~ (2-16)? = 784-216 795 = (784 +2-16) (784-216) 95 * (784-216) = (7844216) +05 = 10+05 = 40. =20 Ans. 05 78A | O. Math vie ela + =3x =: 15 eI o 9. ul 29. 8+7-9- {2:1 - 6:3 - (2:1 + 0-9) + 15-2} = 84+7-9-{2:1-63-3-0+ 15-2} 8+7.9-8 = 79 u 30. Given expression 212 318 x 4lM ah: 10-15 x 535 55 535 265 x 318 = 2x5=10 32. [= | 51 [582-304 1]30 oy 3 QZ 6318 x 212 x DUS y 505 388 x 5-75 7 FS x6 212 54.318 y<(22)M4 »¢ 1018 + 6x we 515 x x 343 x 44S Ble ple x x 3% x 205 2x3x5% 2x3x5% 525 383 x 205 = QOIS+1-GS yx 313 +1-48 yy 57/5- 25 gy 4st BB 5x13 _ 13 15 ~ 23 5 a oa 4 35 4 Ans. Ans. Ans. Ans. 82A | O. Math or, AD-Ax-Bx = C or, (A+B)x = AxD-C . _ AxD-C “ ** “A+B Example 1. If the cost of 30 metres of cloth is Rs. 345, find the eost of 16 metres of this cloth. Solution : Since the cost of cloth’ is to be calculated, hence Rs. must be written to the right hand side Cost of 30 metres of cloth is Rs. 345 Cost of 1 metre of cloth is = 8 Cost of 16 metres of cloth is = A5%16 = Rs. 184 Ans. Example 2. A car completes a journey at 60 km per hour in 5 hours. If the same journey is completed by the car at 5O km per hour, how much time will it take ? Solution : * At 60 km/hr, the car completes the journey in 5 hours «. At 1km/hr, the car completes the journey in 5 x 60 hours 5x60 50 6 hours Ans, . At 50 knvhr, the car completes the journey in Example 3. If 25 men can do a work in 24 days, how many men will finish it in 20 days ? Solution : *. In 24 days the work is complete by 25 men In I day the work is complete by = 25 x24 25x24 20 30 men Ans, In 20 days the work is complete by Example 4. If 4 men or 7 women do a work in 60 days, then in how many days will 8 men and 7 women finish the same work ? 86A | O. Math Exercise Directions—In each of the following questions a number of possible answers are given out of which one is correct. Find out the correct answer. 1. If 6 people spend Rs. 2400 in 4 months then 2 less people will spend Rs. 1800 in how many months ? (A) 4 ®) 6 © 6} @ 44 () None of these 2. If 20 men can complete a work in 3 days while 12 ladies can complete the same work in 12 days, in how many days will 20 men and 20 ladies complete the same work ? 2 5 1 2 A aA ®) 25 © 35 ®) 35 ©) None of these 3. If 4 cows in 2 days eat 18 buckets of grass, then how many buckets of grass will be eaten by 18 cows in 2 days, (A) 81 @) 90 (©) 72 @) 75 €) None of these 4. If 24 men can complete a work in 48 days, then in how many days the work will be completed by 36 men ? (A) 72 @) 32 (©) 18 @) 36 ©) None of these 5. If 10 men or 18 boys can do a work in 15 days, in how many days the double of that work will be done by 25 men and 15 boys ? 9 As (B) 9 (C) 36 @) 18 © None of these 6. If each boy takes two chocolates a day, it will suffice to 7 boys for 12 days. If each boy takes three chocolates a day, then for how many days will suffice to 8 boys ? (A) 10 (B) 15 (C) 16 @ 7 @® None of these 7. If the rate of exchange is 7-50 dollars for Rs. 100, how many dollars will be equal to Rs. 550 ? (A) 41-75 @) 42-75 (C) 41-25 @) 42-25 ©) 4050 8. If the weight of 13 metres long rod is 23-4 kg, what is the weight of 6 metre long rod ? (A) 72kg ®) 108 kg (©) 124kg =D) _—«18-0kg ) None of these 90A | O. Math 35. Nine children take 20 days to complete a work which can be completed by six adults in 15 days. How many days will 5 men and 10 children together take to complete the work ? (A) 9 @) 175 © il @) Cannot be determined ©) None of these 36. If x men working x hours a day for x days produce x articles, then the number of articles produced by y men working y hours a day for y days is— (A) 3/y? @B) xy? (C) yx? @) x/y? (E) None of these (Asstt. Grade Exam. 1991) 37. 16 men complete one-fourth of a piece of work in 12 days. What is the additional number of men required to complete the work in 12 more days ? (A) 48 @®) 36 (C) 30 ©) 16 ®) None of these 38. If 24 workers completes a work in 18 days working 8 hours a day, in how many days will 36 workers complete the same work working 12 hours a day ? (A) 6 @) 8 (C) 10 @) 12 (©) None of these 39. 400 persons working 9 hours a day complete} th of a piece of work in 10 days. The number of additional persons,. working 8 hours a day, required to complete the remaining work in 20 days, is— (A) 675 @) 275 (© 250 @) 225 (©) None of these 40. A work can be done either by 6 men and 5'women in 6 days or 3 men and 4 women in 10 days. In how many days will the work be done by 9 men and 15 women ? (A) 1 (B) 2 (© 3 @) 4 (©) None of these 4]. Mohan engaged a servant on the condition that he would pay him Rs. 200 and a set of clothes after 10 days of his service. The servant left 94A | O. Math 22. 12 AL = oa. No. of more workers to be employed = 50-30=20 Ans. _@ Here, a= 40, Ay=3 ALB A=1,B) =4andB=2 A B 23. Required time = 8 days Ans. ~ aa, — Ala-b) Here, A = 500, a =60 25. Required time = AGB b=12, B=300 500 x 48 800 = 30days Ans. 26. Time taken in covering the distance per day in first case = 24-9=15 hours and time taken in covering the distance per day in second case = 24-10=14hours *’ Time taken in covering 1200 km at 15 hour per day with 1 speed = 70 days .. Time taken in covering 1 km at 1 hours per day with 1 speed 7015 <. Time taken in covering 750 km at 14 hours per day with 3 speed = 20x15 x750x2 ~ 1200 x1x14x3 days Ans. 1 314 98A 1 O. Math 40. In 6 days work is done by (6 men +5 women) 41. 1 day work is done by = 36 men + 30 women In 10 days work is done by (3 men + 4 women) ie 1 day work isdone by = 30 men +40 women oo 36 men + 30 women = 30 men +40 women or, 6men = 10 women 9men = 10 x9 ~ 6 = 15 women 6men+5 women = 15 women and 9men+15 women = 15+ 15 = 30women If 15 women do the work in 6 days 1 woman do the work = 6 x 15 days 30 women do the work = £218 = 3days Ans. Let the price of the set of clothes be Rs. x : Salary for 10 days = Rs. (x+200) and Salary for 5 days = Rs, &*200%5 +200 = Rs. 2 22 x+20 or, x+200 = 2x+40 ws x = Rs. 160 Ans. 102A | O. Math Remaining money after this " 8 t T Money given tohis wife = c% of 100-4). (100-6 W Remaining money after this = (100 - a) (100 — 4) (100 - ¢) 100 x 100 If 100 - a) (100 — 5) (100 - ¢) 100 x 100 is the remaining money then initial money = Rs 100 Ax 100 x 100 x 100 (100 — a) (100 — b) (100 - c) A x 100 x 100 x 100 * (100 -a) (100 — b) (100 - c) 6. If A boys and B girls appeared in an examination and a% of the boys and b% of the girls passed in it, what is the total percentage of students who passed ? Ais the remaining = Total initial amount = Rs. Formula— Axa+Bxb Total percentage of passed students = A+B Proof— Since out of A boys, a % passed axA No. of the boys who passed = 100 Similarly No. of the girls who passed = xB axA ,bxB Total number of students who passed = 00 * 100 _ axXA+bxB ~ 100 axA+bxB._100 % of total students who passed = 100 AGB _ 4XA+bXB A+B 106A 1 O. Math Example 10. ‘The salary of Amit Verma increases by 25%. How many per cent it must be reduced so that it may be equal to the original salary ? Solution : Let the initial salary of Amit Verma be Rs. 100 Increased salary = 100 + 25% of 100 = Rs. 125 Reduction to bring it again to Rs, 100 = 125- = Rs. 25 % in reduction to bring it again initial salary = 25x10 = 20% Ans. By Short-cut Method : Required percentage of reduction Example 11. If the price of a machine is reduced by 20% and the number of machines sold is increased by 40%, what will be the effect on the revenue ? Solution : Let the number of machines sold before be y and the price of a machine before reduction be Rs. x. In that case money obtained = Rs. x.y. Price of a machine after reduction = Rs, And the no. machines sold in second case = _ ar _ 80x Oy Money obtained in second case = 100 * 100 = Rs. 1-12 xy Increase = 1-l2xy-—xy = O12 xy Per cent increase = er 100 = 12% Ans. 110A | O. Math «. Percentage of the first number to the second number = 130x100 140 § = 925 Ans. By Short-cut Method : Percentage of the first to the second 100 + % of the first to the third = 100 + % of the second to the third 100 +30 ~ 100+40 6 ay Example 17. If A's income is 150% more than that of B, then how much per cent is B's income less than that of A ? Solution : Let the income of B be Rs. 100 oe A’sincome = 100+ 150=Rs. 250 If _A’s income is 250 than B's income = Rs. 100 A's income is 100 than B's income = ‘og 2100 = Rs. 40 Difference in A's and B's incomes = 100-40 = Rs. 60 Hence B's income is 60% less than that of A. Ans. By Short-cut Method : Required percentage Example 18. If marks in Mathematics obtained by Veer chand is 10% less than that of Onkar, then how much per cent are marks obtained by Onkar more than that of Veer chand ? 114A | O. Math 18. How much is 20% of 4 of 15407 (A) 44 (B) 55 ©) 88 @) 22 (E) None of these 19. If a cloth-manufacturer allows a discount of 20% on cut-piece cloth and then a discount of 15% to a customer, how much per cent total discount does the customer get ? (A) 38% (B) 35% (C) 32% @) 30% ©) None of these 20. If a man spends Rs. 875 per month and saves 12h% of his income, then what is his monthly income ? (A) Rs. 900 (B) Rs. 950 (C) Rs. 1060 (@) Rs. 1050 ©) Rs. 1000 21. The number of the members of a club is increased by 10% every year. If the initial number of the members is 500, then what will be the number at the beginning of the third year ? (A) 610 (B) 615 (©) 620 @) 605 (E) None of these 22. 49 is 70% of — (A) 700 (B) 200 © 70 @) 490 ©) None of these 23. How much per cent is 20 paise of Rs. 5? (A) 40% @) 20% ©) 04% @) 8% ©) 4% 24. A man earns Rs. 3200. He spends 25% on food and 8% on breakfast. 40% of the rest of the money is spent on rent of the house. If 50% of the remaining money is spent on conveyance then what is his saving ? (A) Rs. 643-50 (B) Rs. 641-20 (C) Rs. 543-20 (D) Rs. 643-20 ©) None of these 25. In an examination 51% of the candidates failed in English while 45% in Mathematics. If 21% failed in both the subjects and 169 candidates passed, then what is the number of candidates who appeared in the examination ? (A) 500 (@) 600 ©) 676 @) 660 ©) None of these 26. If Rs. 85 are added in the income of Pankaj, then the total becomes 50% of Anil's income. What is the income of Anil ? (A) Rs. 540 (B) Data inadequate (C) Rs. 520 (@) Rs. 420 @) Rs. 450 118A | O. Math 13. % effect of revenue = Value of increase % — Value of decrease % __ Value of increase % x Value of decrease % 100 15 x15 = 15-15- 100 = -2:25 .. Since it is negative, hence, there is 2-25% decrease in the revenue. Ans. 14. Let the number of books in the library be 100 _ No. of books in Eng. language = 20 And no. of books in Hindi language = (100-20) x ae No. of. books in other regional languages = 100-—(20 +40) =40 If 40 books in other regional language then total number of books = 100 «. 900 books in other regional language then total number of books _ 100 x900 ~ 40 = 2250 Ans, 15. If 15kgis copper then totalore = 100kg 100 x72 72kg is copper then total ore = 15 = 480 kg Ans, 16. The cost of sugar before the rate of increase was Rs. 6.00, the cost of sugar of same quantity after the rate of increase is Rs. 7-50 ~. Out of Rs. 7-50, the reduction should be of Rs. 1-50 Out of Rs. 100 the reduction should be of Rs. = i x 100 = 20% Ans. rx 100 100-r 25 x 100 100-25 2500 15 = 331% 17. Required percentage Here, r =25 z 1 = 120 _ 18-20% of 7 of 1540 = 1540 x7 x1) = 44 F 122A | O. Math 35. Let the initial price of 1 banana be Rs. x ve No. of bananas purchased for Rs. 10 = 10 And the price of 1 banana after reduction in price = xx ax = Rs. 5 ~. No. of bananas purchased for Rs. 10 at the decrease price _ 10x5 4x _ 25 ~ 2x 25 10 2 x 5 25-20 _ or, = = or, 10x = 5 = Rs * “10 = 50 paise Ans. By Short-cut Method : If the initial price ef one banana be Rs. y, then ye Wo xB (Here, A = 20, x= 10 and B = 5) _ 20x10 * (100-20) x 5 _ 200 ~ 80x5 = Rs} = 50 paise Ans. eg 20 x 100 36. Required increase percentage = 100-20 = 2000 ~ 80 = 25 Ans. 37. Let the number be x 4 3 40% of 5 of 4 of x = 48 or, xt 3 4 = 4 5°47 100 126A | O. Math 5. A persons can complete a work in a days. If B more persons join them after n days of the day they started working, in how many days will they complete the remaining work ? Formula— Required Number of days = At) Proof—Let the remaining work is completed in x days The work done by N persons inn days = ; The remaining work 1 -# =# 7 and the remaining work is done by (A + B) persons -; A persons complete 1 work in a days - (A +B) persons complete a work in axA — Afa-n A+B a 7 A+B °% 6. A takes n times the time in completing a work as taken by B and C together to complete the same work. If all of them can complete the same work in t days, how many days will A take alone to complete the same work ? Formula—Time taken by A to do the work alone = (n+ 1)tdays. Proof—Suppose A takes x days to complete the work .. B and C together will take = days to do the same work Work of A for'l day -t and work of (B +C) for 1 day =? lin 1 lyzll Ee t n+1 1 or, =< x t or, x = (n+ l)tdays. 7. A servant was engaged on the condition that he will get Rs. a per day for work but he will be fined Rs. b per day on being absent. After n days if he gets Rs. c in all, for how many days did he remain absent ? Formula—No. of days of his absence net Proof— Suppose the servant remained absent for x days 130A | O. Math Example 5. Mohan and Mahavir can complete a work in 6 and 7 days respectively. If Mohan starts the work and they work on alternate days, in how many days will the work be completed ? Solution : Work done Mohan in first day 1 6 and work done Mahavir in second day =} Thus work done in 2 days et+t _13 ~ 42. x 13 6 ‘Work done in 6 days =9% > -13 “14 Remaining work after 6 days = 1 =a at “14 Now in 7th day, it is the turn of Mohan ‘ vo ee» and time taken by Mohan to do 14 work = 16% 14 3 = 7 days «. Time taken to complete the whole work =6 +3 = 6 Sdays Ans. By Short-cut Method : Working rule—First of all the product and the sum of the durations of completing the work of both are found after this the product is divided by the sum and quotient and the remainder are found. Then the following formula is used = —2X Quotient + Remainder Number of required days = Dy ration of the completing the work by second man Product = 67 =42 and sum=6 +7 = 13 Quotient = 423 and remainder = 3 n Required days = ax2+3 3 = 65 Ans. 134A | O. Math 8. 12. 13. 14. 15. A can do a work in 9 days. B is 50% more efficient than A. In how many days will B do the same work ? (a) 4 @ 4} © 6 ©)7 @® None of these (Sub-Inspector of Police Exam. 2000) A, B and C together can do a work in 3 days. If A or B alone can do the work in 8 days, then how long will take-C alone to do the same work ? (A) 12days — (B) S days (C) 2days (D) 24 days ©) None of these A can do a work in 9 days, B in 10 days and C in 15 days. B and C together worked for 2 days. If the remaining work is done by A, then how many days does he take ? (A) 13 89 () 10 @) 6 () None of these A and B together can reap a field in 30 days. After 20 days work, B is called away and A takes 20 days more to complete the work. B alone could do the whole work in— (A) 48days (B) SOdays (C) S6days @) 60 days @®) None of these (S.S.C. Asstt. Grade Exam. 1995) X can do a work in 12 days while Y in 18 days. How long will they take if both work together ? (A) 30 ®) 15 © 7% © 7% ©) None of these Ram can do a work in 15 days while Mohan in 25 days. How long will they taken if both work together ? (A) Bedays @) 34deys (©) 9Sdays —D) EEdays ©) None of these Ram and Mohan together can do a work in 6 days. Ram worked for 4 days and left. Mohan completed the remaining work in 9 days. In how many days can Ram alone finish the work ? (A) 12 B®) 14 © 16 @) 15 © 10 Ram, Shyam and Mohan completed a work together for which they got Rs. 63-35. If Ram can complete the work in 2 days, Shyam in 4 days and Mohan in 8 days, then how much money should Shyam get ? (A) Rs. 10-15 (B) Rs. 60-20 (C) Rs. 9-05 (@) Rs. 18-10 ®) None of these 138A | O. Math 4. Time taken by B to complete the work =a ~ ak Here x = 8 and y= 12 8x12 “12-8 = 24 days Ans. 6. Time taken when all the three work together _ xyz “xy + yz + zx Here x = 7, y = 14 and z = 28 _ 7x 14x 28 "7x 44414284287 28 =4 days Ans. 7. Work of X for | day = Work of Y for 1 day = Work of Y for 5days = Remaining work = 1 - Work of X and Y for | day =10 If 7 work is done by both in 1 day 2 work is done by both = 1x 6x 2 = 4days s Required time =2 + 4=6 days Ans. 8. Work of A for | day =3 B will complete the work in 6 days Ans. 9. Time taken to complete the work when they all the three work together ae. “XY HYZ ELE 2 8x Bxz “BxX8+8xz7+z2% 8 __ G4 © 64 + 16z or, 3 142A | O, Math 23. Work of A for 1 day =5— 1 And work of B for 1 day => 30 And work of A¥B4C for 1 day = i 111 Work of C for 1 day =i. 20730724 11 Ratio in their works =20'30' 24 =6:4:5 Sum of ratios = 6+4+5= Share of C “ane 15 = Rs. 12:50 Ans. 24. Let the time taken by them to work together be x hours. Time taken by A =x + 8 hours and time taken by B = x + 4-5 hours 1,1 1 x+8 x+4-5 x ae x+454+2x4+8 1 % (x + 8) (x +45) ox or, 2x2 + 12-5x = x2 + 12-5x 4+ 36 or, x? =36 x =6 Ans. 25. Work of A for 1 day 4 Work of B for 1 day = 1 Work of C for 1 day = Work of B+C for 2 days =2x(i +t == 712 Remaining work Fl-7 at 6*8 L 12 Work of A+B for 1 day = a 146A | O. Math 1 part of the tank is filled = je hours. 3. Three pipes A, B and C can fill a tank in x,y and z hours respectively. If all the three pipes are opened simultaneously how much time will be taken to fill it. _ . .. = xyz Formula: Required time Ba jyerae hours Proof—Part of the tank filled by A in 1 hour = : Part of the tank filled by Bin I hour = ‘ and part of the tank filled by C in 1 hour = : <. Part of the tank filled by all the three pipes in 1 hour etyiyt x yz _ UZ tex txy xyt - YEUX + XY an is filled by (A + B +C) in | hour XYZ ce 1 part is filled by (A+B+C) = Sa eeae hours. 4. A tap fills a cistern in x hours and the other can empty the cistern in y hours. If both the taps are opened simultaneously, how much time will be taken to fill it. Formula— Requiredtime = poe hours Proof—Part of the tank filled by first tap in 1 hour = = and part of the emptied by the second tap in | hour = «. Part of the tank filled by both the pipes in 1 hour eill =o-9 ws xy a > Patt is filled by both the taps in 1 hour i y c. 1 part is filled by both the taps = Fos hours. 150A | O. Math or, cx, 12 2, 1 : 48°16 16 2 18 oe 4816 ax 1 or, 48° «4 x = 4x 8-6 minutes Ans. By Short-cut Method : Required time = yG=2) (Where x = 16, y= 24 and z= 12) _ 24 (16 ~ 12) ~ 16 _ 24x 4 ~ 16 6 minutes Ans. 154A | O. Math 16x 8 16-8 = 16 hours Ans. 4. Since the cistern is already full and the pipe which empties the cistern works faster than the pipe which fills the cistern. Therefore, the same formula will be applied here. 3. Time taken to fill the cistern = Time taken to empty the cistern = s x so 3000 ~ 10 = 300 min. = 5 hours Ans. 5. Time taken to fill the tank = SO RO a ae min. _ 48000 ~ 1200 + 2400 - 800 48000 ~ 2800 = 175 min. Ans. 6. If the waste pipe can empty the cistern in x minutes then 60 = 24x 40x x 24x + 40x - 24x 40 or, = 60x _ , 64x - 960 or, 3840x — 57600 = 960xc or, x = 52600 , 2880 = 20 min. «. Capacity of the cistern = 30 x 20 = 600 gallons Ans. 7. If the waste pipe can empty the full reservoir in x minutes then 15 _ 10 x 12 x x 2 ~ 10x + 12x - 10x 12 or 15 ___120x . 2 22x — 120 or, 165x-900 = 120x or, x= 200 , 45 = 20 min. .. Capacity of the reservoir = 20x 5 = 100 litres Ans. Distance and Time & 9 Questions On Trains Main Formulae : 1, Speed = Distance + Time 2. Time = Distance + Speed 3. Distance = Time x Speed 4, If two trains are going in the same direction, then their relative speed is equal to the difference of their speeds. 5. If two trains are going in opposite directions, then their relative speed is equal to the sum of their speeds. 6. If speed is given in kilometre per hour then in order to convert it into metre per second, it is multiplied by 3 7. If speed is given in metre per second then in order to convert it into kilometre per hour, it is multiplied by - 8. The lengths of two trains are a and b metres respectively. If the speeds of first and second trains be C and D km. per hour respectively and they are running in opposite directions, how long will they take to cross each other ? Formula—Required time in seconds _ sum of the lengths of both trains “sum of their speeds in m/sec Proof— Speed of first train in m/sec = exs + Dx 7g Relative speed = (C+D) x A mneec. and _speed of second train in m/sec and total distance = (a+ b) metre Required time arb seconds. (C+D)x 18 162A | O. Math or, or, y(b-a)=a@ a ba hours. 16. Two trains starts simultaneously from the stations A and B towards each other at the rates of u and v km/hr. respectively. When they meet it is found that the second train had travelled a km more than the first. Find the distance between the two stations. oe y= Formula—Distance between A and B =-4#*) km Proof—Let the distance between the stations be x km. If the first train travelled y km then the second train travelled y + a km. . x=ytyta = 2y+a Since both the trains have taken the same time yta_y v u or, uy+au = or, (v-wy = or, 2) = x= Example 1. A train is running with the speed of 45 km per hour. What is its speed in metre per second ? Solution : Speed of the train in kilometre per hour = 45 ~. Speed of the train in metre per second = 45 x 2 = 12.5 Ans. Example 2. If a motor car covers a distance of 250 metre in 25 seconds, what is its speed in kilometre per hour ? 166A | O. Math and speed = —-- =22 m/sec. = 79-2 km/hr, Ans. Example 8. A student walks from his house at 2 km an hour and reaches his school 6 minutes late. The next day he increases his speed by one kilometre an hour and reaches 6 minutes early. How far is the school from his house ? Solution : Let the distance from his house to school be 1 km Time taken to travel 1 km with the speed of Fm an hour 2 = 1x 5 hour = 60225 minutes ‘Time taken to travel 1_km with speed of 2 km per hour 2 2 hour a2 x 60 minutes 7 __ 120 Difference of time = 24 7 = minutes =9 s 48 ae a a ty is this difference of time, distance = | km 12 is this difference of time, distance = 12x7¢=7km Ans. By Short-cut Method : Distance between the school and the house First speed x second speed Difference of two speeds x Duration between two times 35 Example 9. Krishna covers’a certain distance by train at 25'km/hr and the same distance on foot at 4 km/hr. If the time taken by him for the whole journey be 5 hrs. and 48 minutes, how much total distance did he cover ? 18. 20. 21. 22. 23. 24, 25. O. Math | 173A A train is running at the speed of 36m/secs. If it passes a man walking in the opposite direction at the speed of 4m/sec in 10 seconds, what is the length of the train ? (A) 400 m @) 500 m (CC) 320m @) 360m ® 40m The distance between two stations is 330 km. If a train starts from one station to another at 30 km per hour and another train starts from the second station to the first at 25 km per hour at the same time at what distance from the first station will they meet ? (A) 250km = (B) 150km_ = (C) 80km @) 240 km ©) 210 km Aman walks a certain distance at 8 km/hr and returns at 6 km/hr. If the total time taken by him is 34 hours, the total distance walked is— (A) 28 km @) 24km (© 14km @) 16km (E) None of these (S.S.C. Clerk Grade Exam. 2006) A policeman is running after a thief who has got a start of 200 metres. If the policeman runs ab metres per sec. and the thief at at metres per sec., when will the thief be caught ? (A) 4min. =) 3 min. ~=(©)_2 min. @) 5 min ©) None of these ‘Walking at 4 km per hour a clerk reaches his office 5 minutes late. If he walks at 5 km per hour, he reaches there ab minutes earlier. What is the distance of the office from his house ? (A) 2km ®) 3km (C) 2-5 km @) 4km ©) None of these The time taken by a train 180 metres long, travelling at 42 km per hour, in passing a man, walking in the same direction at 6 km per hour, will be— (A) 18seconds (B) 21 seconds (C) 24seconds (D) 25 seconds ©) None of these (S.S.C. Clerk Grade, Exam. 2006) A monkey is climbing up a greased pole ascends 2 metres and slips 1 Metre in alternate minutes. If the pole is 12 metres high, how long will it take him to reach,the top ? (A) 22min. =) 24min. (C) 25min. @) 22.5 min. ©) None of these The train A is 280 metres long while B is 350 metres long. The speeds of A and B are 38 knv/hr and 25 km/hr respectively. If they are O. Math | 177A If.with 60 km/hr time taken =} hrs 1 km/hr time taken =3x 60 i 3, 60 54 kmhr time taken =4% 64 = or =50 min. Ans. 6. Relative speed = 15 + 12 = 27 km/hr. . Time after which they will meet = 2010 brs. = 1 hr6 min. 40 secs. Ans. Total distance i Speeds Time Total distance 30 = 30 Total distance = 30 x 30 Length of the train Ans. 8. Relative speed = 3045 =35 misec. 150 Time taken to cross a man =35_ =4 2 secs, Ans. 9. Circumference of the wheel =22 x 84 = 264cm Distance covered in 1 sec. = 4 x 264 = 1056 cm , 1056 | 18 Speed in kan/ie = 100 * = 38.016 Ans, 10. Distance of the meeting point fromP , Here, a= 220, b= _ fe-bxt | b = Ubec tty*X c= 100 and => [ze sox t sos} ] go+100 * 21% 10. 11. 12. 13. 14, O. Math | 193A Let the speed of the boat in still water be x km/hr The speed of the boat along the current = (x + 1) km/hr and the speed of the boat against the current = (x — 1) km/hr 4 4 ye. ex-1> a 4(x+1+x- 1) %, “G@tl@-l) 73 or, 8x = 3x7-3 or, 3x7-8x-3 = 0 or, 3x?-9x4+x-3 =0 or, (x-3) Gx+1) = 0 xe 30-4 Hence the speed of the boat in still water is 3 km/hr Ans. Let the speed of the motorboat in still water be x km/hr The speed of the motorboat along the current = x+5 km/hr and the speed of the motorboat against the current = x—5 km/hr 10 10. _ 50 x+5°x-5° 60 10 (x-~5+x+5)_ 5 (4+5)(x-5) ~ 6 or, or, 20x x6 = 5 (2-25) or, x?-24x-25 =D or, x? -25x4+x-25 = 0 oe x = 25 km/hr. Ans. Distance between P and Q 2 ae) (Here, ¢= 3, u=9 and v = 3] U _ 3x (92 - 32) ~ 2x9 = 12 km. Ans. Let the distance of the place from the starting point be x km. The speed of the man along the current = 5 + 1-5 = 6-5 km/hr and the speed of the man against the current = 5 — 1-5 = 3-5 km/hr 24h oy 65735 or, 10x = 6.5% 3-5 x= 275 2.275 km. Ans. Speed of the boat along the current = 1 and speed of the boat against the current = 5 km/hr O. Math | 197A or, ad-x(a+d) = be—x(b+c) or, (b+ Qx-x(at+ ad) = be-ad or, x[(b+c)-(a+d)] = bce-ad be -ad + b+ 0)-(a+d) ee ad —be * (a+ d)-(b+c) Note—If in the question it is asked ‘‘what should be added so that these numbers may be proportional ?’’ then required answer is multiplied by minus sign. 2. In a mixture the ratio of milk and water is a: b. If in this mixture another x litre of water is added, then the ratio of milk and water in the resulting mixture becomes a : c. What are quantities of milk and water in the original mixture ? Formula— Quantity of milk bx c-b and quantity of water = Proof—Let the quantity of the original mixture be m litres ; : Quantity of milk and qiantiy ce wae ew ‘When another x litres of water is added then Jot bm, oe aig at+bat+b . o, _ bm x(at b) a+b = aie a+b a+b % —_am____a@ * bm+x(a+b) ~ ¢ or, cm = bm+x(a+ b) or, m = Hath) a ay _ —ax(a+b) Quantity of milk = C-hath litres a). = > —piitres 2 bx. and quantity of water = —— litres. c-b O. Math | 201A i Mohan and Rakesh = =4:3 Ans, Example 6. ‘What should be subtracted from 15, 28, 20 and 38 so that the remaining numbers may be proportional ? Solution : _20-x Ifx is subtracted then 2 x *ago5 or, (1S5—x) G8-x) =(28-x) (20-2) or, x2 —53x4570 =x? - 48x + 560 or, xx? -53.x4+48x = 560-570 or, 5x =10 or, x =2. Ans. By Short-cut Method : t ad~be Here, a= 15, b=28 Required number = Gy —(b+c) c= 20and d=38 _ _15 x38-28 x20 ~ (15 + 38) — (28 + 20) _ 510-560 ~ 53-48 = 0 a) =2 Ans. Example 7. If Rs. 279 were distributed among Ram, Mohan and Sohan in the ratio of 15: 10:6 respectively, then how many rupees were obtained by Mohan ? Solution : nat in which Ram, Mohan and Sohan got = 15: 10:6 Sum of ratios = 15 +10+6 =31 Share of Mohan = 10n2P =Rs. 90 Ans. 10. 1. 12. 13. 15. 16. O. Math | 205A Marks obtained by Komal in English are equal to 1/3rd of marks obtained by her in music. Total marks obtained by her in both the subjects together is 160. How many marks did she secure in music ? (A) 120 @B) 60 (C) 30 @) 90 (©) None of these If the ratio between the price of scooter and television is 3 : 2 and the price of the scooter is more than the price of the television by Rs. 6000. What is the price of the television ? (A) Rs. 18000 (B) Rs. 10000 (C) Rs. 12000 (D) Rs. 6000 ®) None of these If Rs. 270 is divided between A, B and C such that A's portion is equal to 24 of B's portion and B's portion is equal to 3rd of C's portion. What is the share each of A, B and C ? (A) Rs. 90, Rs. 60 and Rs. 120 (B) Rs. 120, Rs. 60 and Rs. 90 (© Rs. 90, Rs. 120 and Rs. 60 (D) Rs. 60. Rs. 120 and Rs. 90 ©) Rs. 60, Rs. 90 and Rs. 120. If Rs. 90 is divided between A, B and C in the ratio of 2 : 3: 5. Whatis B's share ? (A) 3 (B) 26 (C) 25 @) 27 (BE) None of these The weight of the mixture of water and milk is 35 litres. The ratio of water and milk in it is 1 : 4. If some more water is added in the mixture then its ratio is 1 : 2. What is the amount of water added ? (A) Glitres (B) Slitres (C) 7litres @) 4litres (©) None of these If the profit of Rs. 6000 is divided between A, B and C in the ratio of 4:5: 3, what is the difference between the shares of B and C ? (A) Rs. 500 (B) Rs.1200 (C) Rs.2500 (@) Rs. 400 ©) None of these Anand and Gopal together have Rs. 121. If ded of Anand's money is equal to : th of Gopal's amount, what is the amount that Gopal have ? (A) Rs. 66 (B) Rs.60.50 (C) Rs.26.40 (D) Rs.55 © None of these Mihir, Chetan and Pinto hired a car for Rs. 2600 and they used it for 24 hours, 36 hours, and 44 hours respectively. The amount of hire, charged from Chetan was : (A) Rs.600 (B) Rs.1100 (C) Rs.900 @) Rs. 866.66 (®) None of these 40. 42. O. Math | 209A = if = 3, the ratio of 2x + 3y) and (3y ~ 2x) is— (A) 2: @) 3:2 © 3:1 @) 21:1 ©) None of these x y Five mangoes and four oranges cost as much as thrée mangoes and seven oranges. What is the ratio of the cost of one mango to the cost of one orange ? (A) 4:3 @) 1:3 (C) 3:2 @) 5:2 ©) None of these (Sub-Inspector Police 2007) Annual incomes of Amit and Veeri are in the ratio of 3 : 2 while the ratio of their expenditures are 5 : 3. If at the end of the year each saves Rs. 1000, what is the annual income of Amit ? (A) Rs.9000 (B) Rs. 8000 (C) Rs.7000 (D) Rs. 6000 @) Rs. 5000 Answers 1.(A) 2@) 30) 4@8) 5.8) 60) 7.© 8) 9.(A) 10.(C) 11.) 12M) 13. 14@ 15.0) 16© 17.(B) 18.(A) 19.(C) 20.2) 21.@) 22.(C) 23.) 24.(D) 25.(C) 26.(B) 27.(A) 28.(C) 29.) 30.@) 31.(C) 32.© 33.(D) 34.) 35.(D) 36.(C) 37.©) 38.(A) 39.(C) 40.0 41.(C) 42.) Hints . Ratio between the first and second number = 2: 3 and Ratio between the second and third number = 5 : 8 To make the ratio of the second number, same in both multiply by 5 and 3 respectively. .. Ratio between the first, second and third number Ratio between the three numbers = Le, =23 =2:3:4 O. Math | 213A 14. Sum of ratios =44+5+3 =12 And difference in the ratios of B and C =5-3=2 Difference between the shares of B and C _ 2x 6000 ~ 12 =Rs. 1000 Ans. 15, x part of Anand’s money = 2 part of Gopal's money Anand's money =3 x part of Gopal's money =$ part of Gopal's money Ratio in Anand's money and Gopal's money =6:5 Sum of ratios Gopal's money = is 121 =Rs. 55 Ans. 16. Ratio of hours used by Mihir, Chetan and Pinto = 24:36:44 =6:9:11 => Charge from Chetan = z 2600 =Rs. 900 Ans. 17. Ratio of metal A and B is 13:7 Amount of metal B in 100 kes. = 2 x 100 = 35 kg. Ans. 18, Ratio of two watches = 16:23 When cost of first watch = 16x then cost of second watch = 23x 16x+1.6x 11 x+447 ~ 20 352x =253x +4917 90x = 4917 4917 99 _ 449 ~ 3 uy > p= O. Math | 221A | «Share of Bin the profit = See =Rs. 5200 Ans, By Short-cut Method : Share’of B in the profit = Rs, 2 pfere, Cy = 8000, C; = 10000 and P = 9360] Cy +Cy _ 10000 x 9360 » 8000+ 10000 = Rs. 5200 Ans. Example 2. A and B enter into a partnership. A contributes Rs. 5000 while B contributes Rs. 4000. After 1 month B withdraws ; part of his contribution and after 3 months from the starting A puts Rs. 2000 more. When B withdraws his money at the same time C also joins them with Rs. 7000. If at the end of | year there is a profit of Rs. 1218, what will be share of C in the profit 7° Solution : Since the contributions of three partners are different and their times also differ. Therefore, their contributions should be converted for equal durations. For this, contribution is multiplied by time. ’. Contribution of A = Rs. 5000 for 12 months + Rs. 2000 for 9 months Contribution of A for 1 month= 5000 x 12 + 2000 x9 = 60000 + 18000 = Rs. 78000 Contribution of B_ = Rs. 4000 for | month + Sof Rs. 4000 for 11 months Contribution of B for 1 month= 4000 x 1 +3000 x 11 = 4000 + 33000 = Rs. 37000 Contribution of C = Rs. 7000 for 11 months Contribution of C for 1 month = 7000 x11 =Rs. 77000 Ratio in their contributions = 78000 : 37000 : 77000 ‘ = 78:37:77 Sum of their ratios = 78 + 37 +77 = 192 > Share of C in the profit. = 7/2218 = Rs, 488.47 Ans. 8. 11. 12. 13. 14, O. Math | 225A Ram started a business with Rs. 3400. Three months later Bhagwan joins and a further 3 months later chandu joins them. What is the amount of capital put by Chandu if at the end of the year their shares of profit are as 2:3:57? (A) Rs. 8500 (B) Rs. 17000 (C) Rs.5200 (D) Rs.5100 ©) None of these A and B enter into a partnership with capital in the ratio of 5 : 6. After 8 months A withdraws his capital. If their shares in profit are in the ratio of 5 : 9, then for how much time the capital of B remained in the business ? (A) 12months (B) 10 months (C) 16months (D) 18 months (E) None of these Prabhat's share in a business is? th of Prakash's share. If Prakash's profit is Rs. 1200 and his investment is Rs. 40000, what is the ratio of Prabhat's profit to his investment ? (A) 100:3 @) 3:10 (C) 10:3 @) 3:100 ©) None of these Suresh started a business with a capital of Rs. 12000. Four months later Dinesh joined him with a capital of Rs. 7000. If at the end of the year total profit earned was Rs. 13300, what is the share of Dinesh in the profit ? (A) Rs.9576 (B) Rs.4900 (C) Rs.8400 (D) Rs. 2756 ©) None of these ~ Rahim, Karim and Shamim invest Rs. 2000, Rs. 3000 and Rs. 4000 respectively in a business. Rahim withdraws his capital after 4 months and karim withdraws his capital after 8 months. If at the end of a year the profit is Rs. 900, what is the share of Rahim in the profit ? (A) Rs.270 = (B) Rs. 180 = (C)_- Rs. 540 @) Rs.90 (E) None of these Avadh, Bipin and Suresh enter into a partnership. Avadh puts in Rs. 2000 for the whole year. Bipin puts in Rs. 3000 at first and increases it to Rs. 4000 at the end of 4 months. Suresh puts in at first Rs. 4000 but withdraws Rs. 1000 at the end of 9 months. If at the end of the year the profit is Rs. 847.50, what is the share of Suresh in the profit ? (A) Rs. 180 (B) Rs.330 (C)_ Rs. 337.50 @) Rs. 282.50 ®) None of these Ram, Shyam and Mohan hired a field for Rs. 500 to graze the cows for the whole year. Ram puts in 15 cows for 6 months, Shyam 12 cows for 5 months. If Mohan puts in 25 cows for 4 months, what is the share of Mohan in the rent ? (A) Rs. 180 (8) Rs.120 (C) Rs. 250 @) Rs. 240 (©) Rs. 200 O. Math | 229A [Here, c, =25, 1 =6, c =40,t9 =8, cy = 30, f3 = 12, and P= 415] 30x 12x 415, * 75 %6440 x8 +30 x12 149400 150 + 320 + 360 _ 149400 830 = Rs. 180 Ans. 5. Share of Kumud in the profit Cat, XP * cit + ot [Here, c, = 12000, t; = 12, cpt = 16000 x 6 + 8000 x 6 and P = 4200] _ _ (16000 x 6 + 8000 x 6) x 4200 ~ 12000 x 12 + 16000 x 6 + 8000 x6 _ 144000 x 4200 ~ 144000 + 144000 = Rs. 2100 Ans, 6. Share of A for looking after the business = 15% of Rs. 5200 =Rs. 780 Remaining profit = 5200-780 = Rs, 4420 +3 2500x4420 Share of A for his investment = 250047500 =Rs. 1105 Total share of A = 780 + 1105 =Rs. 1885 Ans. 7. Contribution of Ram for 5 months = Rs. 5000 ‘ Contribution of Ram for 1 month = 5000 x5 = Rs. 25000 Contribution of Shyam for 6 months = Rs. 6000 Contribution of Shyam for 1 month = 6 x6000 = Rs. 36000 Ratio in their contributions = 25000: 36000 = 25:36 2 25 x 610 Share of Ram in the profit = 25 + 36 Rs. 250 Ans, 30. O. Math | 233A . Share of C in profit AXP [Here, c) xt) = 3512420 x 2+ 2412420 1 = er Aa 2 Xty = 4222180 x 3 c3 Xf = 4065400 x 2 Mahala bet + 4865400 x 1, and P= 1053000] — —(4065400 x 2 + 4865400 x 1) x 1053000 ~ (3512420 x 2 + 2412420 x 1) + 4222180 x 3+ (4065400 x 2 + 4865400 x 1) = Rs. 389886 Ans, . Since Kishan's investment is thrice the investment of Nandan and period is two times the period of investment of Nandan, hence ratio of the investment of Kishan and investment of Nandan = 6 : 1 Total profit =1 5 4000 =Rs. 28000 Ans. Share of Amit in profit _ _&oty xP (Here,c, = 75000, ¢, = 12, cz = 112500 ~ Qty + Coty b =8 and P= Rs. 22500] — — 112500 x 8 x 22500 ~ 75000 x 12 + 112500 x8 Rs. 11250 Ans, O. Math | 237A Solution : Since there as many trees in a row as there are rows, hence the total number of rows will be the square root of 5776. 7 6 1) 57 76 [49__ 146] 876 876 x a No. of rows = 76 Ans, Example 6. What is the cube root of 1000 ?. Solution : 1000 2x2K2 «x Sx5x5 Cube root of 1000 «5 =10 Ans. Example 7. What is the square root of 1 a Solution : 125. 169 144 ~ 144 169 =13 and V144 =12 25° 13 Square root of 1 144 “12 wie =1 12 Ans, Example 8. What is the smallest number that must be subtracted from 2361 in order to get a perfect square ? Solution : 4 8 4/2361 16 88| 761 704 57 «. If 57 is subtracted from 2361 the result will be a perfect square. Ans. O. Math | 241A (Vt0-Va) (Vi0+Va ) Vi0+V8 10~8 * To+ve - —2 _ ” Fiosve ge, WIN) ieV8) Mia <= Tele oe lS Vi4V5 — © VT4NS Out of these three, numerator in each is same but the denominator of the third is the least. Since in case of fractions if numerators are same then the fraction having the denominator of least value is the greatest. Hence Vi-Vs has the greatest value. Vi0-V8 = 26. 27. 28. 29. 30. 31. 32. 33. O. Math | 245A If the approximate value of ‘V 24 is 4.898, then V Bis nearest to— (A) 0.544 (B) 1.633 (C) 2.666 @) 1.333 (E) None of these @.O. Exam. 2008 What is the smallest number that must be subtracted from 438867 to get a perfect square ? (A) 623 @) 640 (C) 665 ©) 633 ) None of these (A.A.0. Exam. 2007) If a=9 and b= 16, then the value of Vab+ 07 is— (A) 20 (B) -20 (C) +20 @) +25 ©) None of these The standard form of V 1.21 x 10° is— (A) 11x10) 11x10? (©) 11x103 () 1.1104 ) None of these rg oom 1+ (A) 60-2 V899 (@) 30+ V899 1 (C) 30-V899 OQ >—-= 36 -V 899 ©) None of these The cube root of 15635 lies between— (A) 20and30 (B) 30and40 (C) 40and50 (D) 200 and 300 (©) None of these The square root of 0.0036 is— (A) 06 ) 0.06 (C) 0.006 (@) 0.0006 (E) None of these The expression 1 — —., — 1+ v2 1-V2 (a) 22-1 @) 1-22 ©1-V2. @ 2V2 (E) None of these is equal to— O. Math | 253A Formula—Average age of going persons = a + ( -“) b years. 3. The average of N persons is a years. If n persons join them, the average age is decreased by b years. What is the average age of new comers ? Formula—Average age of new comers = a — (: + “ years. | 4. The average age of N persons is a years. If n persons leave them, the average age is decreased by b years. What is the average age of going | persons ? Formula—A verage age of going persons = a — ( 1 ) b years. 5. In a group of N men whose average age is increased by b years when aman of a years is replaced by a new man. What is the age of new comer ? Formula—The age of new comer = a + Nb years. Proof—Let the average age of previous N men be A years and the age of new comer be x years Total age of Nmen = N. A years ~. Total age of N men when | man of a years is replaced by a new comer of x years = N. A—a + x years Bus this time the average age is increased by b years. New average age = A+ years NA-a+t N(A+4) or, NA-a+ NA+Nb or, x = a+Nbyears. 6. The average temperature for Monday, Tuesday and Wednesday was F°C. The average temperature for Tuesday, Wednesday and Thursday was L°C. If the temperature for Monday was f° C. What was the temperature for Thursday ? Formula—If the temperature for Thursday was /°C, then f-l = n(F-L) Erealeaney temperature for Monday, Tuesday and Wednesday = .. Total temperature for Monday Tuesday and Wednesday = 3 F ** Average temperature for Tuesday, Wednesday and Thursday = L® «. Total temperature for Tuesday, Wednesday and Thursday = 3 x L Total temperature for Tuesday and Wednesday = 3F —f .. Temperature for Thursday = 3 L - 3F —f) or, l= 3L-3F4f or, f-1 = 3F-L) n(F-L) [Here, 2 = 3 the number of days] 7. A car travels from one place to another at the speed of a km/hr but returns at the speed of 6 km/hr. What is its average speed during the whole journey ? O. Math | 257A and total distance covered in the entire journey =16+20+10 =46km ~. The average speed for the entire journey 46 23 ‘Awe, Example 4. Avehicle travels from Ato B at the speed of 40 km/hr, but from Bto A at the speed of 60 km/hr. What is its average speed during the whole journey ? Solution : Let the distance from A to B be x km Time taken from A to B = 75 brs time taken from B to A = gs Total time taken for the whole journey -= 2 =40* 60 _3xt+2x ="120 = = 120 x = 74 hours and total distance covered for the whole journey = x+ x=2xkm 2x Average speed = WDA _2e x24 x = 48 knvhr. Ans, and By Short-cut Method : Average speed = O. Math | 261A Solution. In such question to find out the average of new numbers, 3 is subtracted from the given average 12. Hence average of new numbers is 9. Ans. Example 12. Out of three numbers, first number is twice of the second and thrice of the third number. If the average of all the three numbers is 44, what is the difference of the largest and the smallest numbers ? Solution. Let the first number be x then, second number = ; and third number = 3 17,4242) = 3 (+343) = 4 or, Setaa tae = 44x3 or, lx = 44x3x6 So 44 x3 x6 - 1 = Greatest number = 72 andthe least number = 72 x3 24 Required difference = 72-24 By Short-cut Method : First number = Least number Read. difference 3 abx a+b+ab 3x2x3x 44 34+24+3x2 72 (Here, a = 2,6 = 3 and x =44] 1 an 3x 24 72-24 48 278A | O. Math _ 2ab? x 100 x (10000— b*) ~ (10000 — b?) (1004) (100 + b + 100- b) 2ab? x 100 100a x 200 _ (4) ~ (ic) 3. How much per cent above cost price must the marked price of an article be marked to make a profit of b% after allowing a discount of a% ? Formula— _— Required percentage = a+ x10 Proof—Let the cost price of the article be Rs. 100 S.P. for b% profit = Rs. (100 + b) If the marked price is Rs. 100, then S.P. after allowing a discount of a% = Rs. (100— a) If Rs. (100 — a) is S.P. then MP. = Rs. 100 Rs. (100+ 5) is SP. then MP. = 22-000+4) 100-a v Difference between M.P. and C.p, = 1000+ 2) 00+ 100 _ 10000 +1005 — 10000 + 100a = (100—a) _ 100(a+b) ~ 100-a . _ 100(a+6)_ 100 Required percentage = “a00-a) *700 _ 100(a+5b) ~ 100-a 4, If two successive discounts on an article are r,% and r2% respec- tively, what is the single equivalent discount on it ? x Formula—Single equivalent discount = r, + ro aa 3 Frock=Let the marked price be Rs. 100 S.P. after the first discount = Rs. (100-;) 1 and discount at r2 % on Rs. (100- r;) = Rs. a (100 - r, Jry 100 100r) + 1007 -ryr2 * 100 Xt 100 Single equivalent discount n+ ntn- 282A | O. Math Proof—Let the C.P. of 1 kg of goods be Rs. 100 CP. of agm of goods = 100 x2 2 to00 = 79 S.P. of a gm. of goods = Rs. 100 . a Gain = 100- 10 1000 -a 10 1000-4 % 100 10 allO (1000 —a) x 100 a 12. A person brought two articles for Rs. p. On selling one article at A% profit and other at B% profit, he gets the same selling price of each. Find the cost price of each article. = Rs. Gain % wa. — pe (100+B)p Formula— C.P. of first article = Rs. 200+A4B AaB can = pe (100+A)p and C.P. of secondarticle = Rs. 599, 44 B Proof—Let the C.P. of first article be Rs. x i CP. of the second article = Rs.(p—x) Rs, 200+) "100 Rs, P= #)(100 +B) 100 S.P. of first article at A% profit and S.P of second article at B% profit Since S.P's of both articles are same x(100+A) _ (—*x) (100 +B) 100 100 or, x(100+ A) = (p—x) (100 +B) or, x(100+A)+ x(100+B) = p(100+B) or, x(100+A+100+B) = p(100+B) _ p(100+B) o * = RS O00+A+B) and CP. of second article = Rs. p-x _ p(100 +B) Rs [p 200+A+B = Rs [eee em EO Bi “ 200+ A+B Rs p(100+A) "200+A+B O. Math | 291A , Example 16. i If the cost of 33 articles be the same as the selling price of 30 articles, what is the percentage of profit ? Solution : Let the C.P. of | article be Re. 1 we CP. of 33 articles = Rs. 33 and S.P. of 30 articles = Rs. 33 33x33_,, 363 S.P. of 33 articles = 30 =Rs. 10 Profit on Rs. 33 = 383 53 = Rs, 2 = Rs. 10 é = 33,100 Profit on Rs. 100 = 10% 33 = 10% Ans. By Short-cut Method : If the cost of x articles is the same as S.P. of y articles, Then Profit % = a 100 Example 17. A dishonest shopkeeper professes to sell goods at his cost price but uses a false weight of 950 gms. for each kilogram. Find his gain per cent. Solution : Let the cost of 1 kg. of the goods be Rs. 100 The ccst of 1 kg of the goods be = Rs. 100 100 x 950 1000 Rs. 95 «. He sells the goods of Rs. 95 in Rs. 100 Gain = 100-95=Rs.5 Gain % = 2m Cost of 950 gms. of the goods = a 1 8 Ee Ans, 6 298A | O. Math 33, 35. 37. 38. 39. The marked price of an umbrella is Rs. 80. If it is sold for Rs. 68, what is the rate of discount ? (A) 7th (B) 20% (C) 12% @) 15% (E) None of these Aman sold his watch at a loss of 5%. If it would have been sold by Rs. 56.25 more, then he would have gain 10%. What is the C.P. of the watch ? (A) Rs. 295 (B) Rs. 335 (C) Rs.375 (D) Rs. 400 ) None of these A trader bought some locks at the rate of 8 locks for Rs. 34 and sold them at 12 locks for Rs. 57. How many lock must be sold by him to gain Rs. 900? (A) 1400 @) 1600 (C) 1800 @) 2000 ©) None of these Bhajan Singh bought 120 reams of paper at 1 ream for Rs. 80. He spent Rs. 280 on its transport and paid 40 paise per ream for octroi duty and the coolie charges were Rs. 72. What should be the S. P. of each ream if he wants a profit of 8% ? (A) Rs.90 —-(B) Rs. 87.48 (C) Rs. 89 () Rs.86 ) None of these There are two successive discounts of 35% and 10% respectively on a saree. If a man has given Rs. 1170, what was the price of the saree before the discount ? (A) Rs. 1800 (B) Rs.1700 (C) Rs.2000 (D) Rs. 1900 (E) None of these A man sells each of his two articles for Rs. 99. On one he loses 1% and on the other he gains 10%. In the entire transaction he gains— (A) 9% B) 44% © 45% @) 55% ©) None of these The cost price of 16 articles is equal to the selling price of 12 articles. The gain per cent is— 33h BOR 25 ©) 75 (E) None of these 302A | O, Math Actual gain = 580 - 400 =Rs. 180 180 x 100 Profit% = 400 =45% Ans, 9. Let the C. P. of radio be Rs. 100 C.P. forDitip = 3. 100 =Rs. 75 ‘ and S. P. for Dilip = 100 + 20 =Rs. 120 = Ans. 10. Insuch questions there is always loss. Total loss = ( oR ; (Here, a = 1540 and b = 12] b _ 21540 = 7100); _ ; 12 _ 3080 ©, 10000 | 1 144 _ 3080 x 144 10000 — = Rs. 45. Ans, ML. S. P. of the first horse = sop (100415 =Rs. 575 C. P. of the both horses =2 x 500 =Rs. 1000 S. P. of the both horses = 1000-45 =Rs. 955 S. P. of the second horse = 955 —575 =Rs. 380 Ans, _ (10 + Profit %) x 100 12. 40 = 100 -10 O. Math | 313A S.L for xyears = 100-100 = Rs.(n- 1) x 100 (a—1) x 100 x 100 100 xx = (n=1) x 100, x Rate = 5. What annual payment will discharge a debt of Rs. A due in years at r% per annum ? 100A nt 1001 + ae Formula—Annual payment = Rs. Proof—Let the annual payment be Rs. x Since the first instalment is paid at the end of first year (t- 1) xrxx 100 (t=2) xrxx 100 ~. Amount of first instalment at the end of ¢ years = x + Similarly amount of second instalment at = x + Thus, total amount of x instalments, _ epee) [ (adare] A= [+ 100 + ]xt 100 i = = &+ Fo le D+(t-2)+...41) m [t--1) xt = ati L 2 J or, 100 wre {2D Y ~ 1004 or, x [100 +249] = pe 6. The rate of interest for first r, years is r)% p.a., for the next fy years is r2% p.a. and beyond this it is 73% p.a. If the simple interest for t3 years is Rs. a, what is the principal ? 100 xa Formula—Principal = Rs. errr area Proof—Let the principal be Rs. x xt x S.L. for 4; years on Rs, x atr;% = Rs. “a XxX Rs, 222%” S.L for t years on Rs. x at r2% 100 320A | O. Math By Short-cut Method : 100 xA [He A=6450 100 xt +e. Annual instalment t=4,r=5 _ __ 100 x 6450 100 4.45% 423 Rs. 1500 Example 11. Divide Rs. 2600 in two parts in such a way that the interest on the first part at 5% per annum for 5 years is equal to the interest on the second part at 4h per annum for 6 years ? Solution : Let the sum of money at 5% be Rs. x F Second part = Rs. (2600 — x) XxX5SX5 _ (2600 -x) x45 x6 100 100 or, 25 x =70200-27 x or, 52 x = 70200 = 20200 52 =Rs. 1350 and other part = 2600 - 1350 =Rs. 1250 Ans. Example 12. The rate of interest for the first 2 years is 3% per annum, for the next 3 years is 6% and beyond this, it is 9%. If the simple interest for 10 years is Rs. 2760, what is the principal ? Solution : Let the principal be Rs. 100 iG S.L on Rs. 100 for the first 2 years =2 x3 Rs. 6 S.1. on Rs. 100 for the next 3 years =3 x6 =Rs. 18 x5 Rs. 45 +18445 =Rs. 69 and S.I. on Rs. 100 for the further next 5 years Total S. 1. 18, 19. 21. 22. 24. 0. Math | 327A A sum of money at simple interest amounts to Rs. 744 in 2 years and Rs. 816 in 3 years. The principal is— (A) Rs.600 (B) Rs.625 (C) Rs. 675 @) Rs. 700 (E) None of these Rs. 600 amounts to Rs. 720 at simple interest in 4 years. If the rate of interest is increased by 2%, what will be the amount ? (A) Rs.648 = (B) Rs.672. (C) Rs. 768 @) Rs. 792 (©) None of these What sum of money will amount to Rs. 1460 in 4 years at ids per annum simple interest ? (A) Rs. 564 (B) Rs. 1000 (C) Rs.790 @) Rs. 1200 (E) None of these If a certain sum of money at simple interest amounts to Rs. 2800 in 2 years and to Rs. 3250 in 5 years, what is the rate of interest per cent per annum ? (A) 5% (B) 3% (©) 4h% ©) 6% (E) None of these What is the simple interest on Rs. 540 at 3% per annum in 24 years ? (A) Rs. 32.50 (B) Rs. 59.50 (C) Rs.40.50 (D) Rs. 27.50 ©) None of these In how many years Rs. 600 at 10% will earn the same simple interest as Rs. 800 at 12% in 5 years ? (A) 12 years (B) 6 years (C) 16 years @) 115 years €) 8 years Hira Lal invested Rs. 500 for 4 years and Rs. 600 for 3 years at simple interest. If the total interest on both is Rs. 190, what is the rate of interest per cent per annum . (A) 10% (B) 2% © 7% ©) 5% () None of these A money lender borrows some money at 4% simple interest and pays its interest at the end of the year. He lends this money at 6% simple interest and gets its interest at the end of the year. If his profit in a year is Rs. 104.50, what sum of money does he borrow ? (A) Rs.5000 (B) Rs. 2090 (C) Rs.5225 (D) Rs. 4000 (©) None of these If a sum of money doubles itself in 16 years, how many times will it be in 8 years ? 1 1 A 1, ®) 1, © 1 ©) None of these 1 1 3 © 16 338A | O. Math 17 Compound Interest Money is said to be lent at Compound Interest (C.1.) if the interest is not paid as soon as it falls due but is added to the principal after a fixed period, so that the amounts, at the end of the period becomes the principal for the next period. If A is the amount, and C.I. is the compound interest, then R\t A =P(1+5 ci. (0% | Note I—If the moncy is payable half yearly time is doubled, rate is being halved i.e. if Rs. 1000 is borrowed at C.1. at 6% per annum for 2 years payable half yearly then for R we use 3% and T= 4. Note I—Compound Interest for one year is equal to the simple interest for one year. Some Other Formulae : 1. What is the difference between compound interest and simple interest on Rs. P for 2 years at r% p.a. ? . r\ Formula—Difference = Rs. P G 00 ) px2xr Rs 100 and C.l. on Rs. p for 2 years at ro =Rs.P[ (1 + iw) =] «. Difference between C.I. and S.I. P [Cre pp 1+ 2e4 oe =? (im) 2. What is the difference between compound interest and simple interest on Rs. p for 3 years at r% p.a. ? 5 _r \2 (4 Formula—Difference = Rs. P(;55 ) Got?) Proof—S.I. on Rs. p for 2 years at r% = " 3424 | O. Math lee f ' a= (ue r log p = alog (14755 r and logq = tlog (1 * Too logpp _@ logq — or, ts S108 4 years log p 8. Jf asum of money put out at compound interest amount in A years to Rs. x and in B years to Rs. y, find the rate of interest per cent per annum. Formula— Rate % = [Q) m1] Where, n = B - A and x is an integer Proof—Let the principal be Rs. p and the rate be r% p.a. r A P(1+i% Pe (1+) 100 —~_\p y _ (06) =~ (+. A 100, (4g) 100, eye AH, r 1+ 100 te Q@*-1 100 \x, or, 1% = [@"- 1] x 100 9. Two persons together lend Rs. p at r% compounded annually. The amounts which first gets at the end of a years is the same as the other gets at the end of b years. Find the share of each in the principal. Pp Rs. 1. (10) 100 y iy * lows +r, P i 100_\ °-« e a) x and y u or, Formula— Share of first person and share of second person = Rs. | p — 348A | O. Math 1261 =Rs. “80 100 x5x3 100 =Rs. 15 . 1261 Difference = 80 -15 61 80 If Rs. St is the difference then P =Rs. 100 and S.I. for 3 years = =Rs. on «. Rs. 11.40 is the difference then P= —~——~ x 11.40 =Rs. 1495 Ans. By Short-cut Method : Difference between C.I. and S.I. for 3 years Peover = iam (ies *?) wens nao =PX25 (5.43) 11.40 x 400 x 100 = 305 =Rs. 1495 Example 9. If a sum of Rs. 13040 is to be paid back in two equal annual instal- ments at 3 3 per annum, what is the amount of each instalment ? Solution : Let each instalment be equal to Rs. x Amount after 1 year = 13040 (4; + _ 13040 x83 ~ 80 =Rs. 13529 Money left after first instalment = Rs. (13529 - x) ai 14. 15. 16. 18. O. Math | 353A. If a certain sum of money invested at compound interest amounts to Rs. 1452 in 2 years and Rs. 1597.20 in 3 years, what is the rate of interest : (A) 11% (B) 12% (C) 10% (D) 9% (©) None of these If Rs. 500 amounts to Rs. 583.20 in 2 years compounded annually, the rate of interest per annum-is— (A) 6% (B) oh% (C) 7% @) 8% (©) None of these Ram and Shyam together lent Rs. 17261 at 5% interest compounded annually. If the amount of Ram for 2 years is the same as the amount of Shyam for 5 years, then Ram's investment is more than Shyam's investment by— (A) Rs. 8000 (B) Rs.9261 (C) Rs.4000 (D) Rs. 1261 (©) None of these A certain sum of money invested at compound interest for 2 years at 20% per annum. If the interest is added half yearly instead of per annum, then the interest increases by Rs. 482, what is the sum of money ? (A) Rs. 50000 (B) Rs. 30000 (C) Rs. 20000 (D) Rs. 10000 ©) None of these If Rs. 1000 is lent at the rate of 10% per annum at compound interest then what will be its amount after 3 years ? (A) Rs. 1111 (B) Rs. 1221 (C) Rs. 1470 =) _ Rs. 1500 @ Rs. 1331 If the simple interest on a certain sum of money at 5% per annum for 2 years is Rs. 160, what will be the compound interest on the same sum for the same time and the same rate of interest ? (A) Rs. 146 (B) Rs. 168 = (C)- Rs. 165 @) Rs. 164 ©) Rs. 170 If a certain sum of money invested at compound interest amounts to Rs. 8820 for 3 years and Rs. 9261 for 4 years, what is the rate of interest per annum ? (A) 4% (B) 3% (C) 5% @) 6% (E) None of these In how many years will Rs. 500 amount to Rs. 605 at 10% per annum compound interest ? (A) 3 years (B) 2 years (C) 4 years @) 2.5 years (E) None of these If a certain sum of money amounts to Rs. 800 for 2 years and Rs. 880 for 3 years then how much will it amount in 4 years ? (A) Rs.920 (B) Rs.968 (C) Rs.898 (D) Rs. 1000 ©) None of these 2 d =P(i5) o-1() P 50 x 100 x 100 4x4 =Rs, 31250 eee log q 3. wired time = 2°84 logp 4 log 8 log 2 4 log23 log2 = 4x3 Jog? log2 = 12 years. 4. Whenn = 3, then 3 xa 3+ oe 4 Gy __ 3xka “00 * Cito)" _ _3a ~ 3-1216 _ 303-60 x 3-1216 3 Rs. 315-90 SI = 303-60 = . #\2 5. When n =2, difference = P(;55) 10 15 = PS or, P= 15 x 100 x 100 , 10 x 10 = Rs. 1500 6. Half yearly rate of interest =4% O. Math | 357A Ans, [Here, a= 4, p=2 and q=8] Ans. a= compound iiiterest [3 S.L = 303-60 and r = 4% Ans. Here, difference = Rs. 15 r= 10and P = principal andtime = 2 (half years) CL = 15000 [C+ i6 ra] O. Math | 363A 27. Amount after 3 years 5\3 5\2 =200 (1+ 355 +200 (1+ 595 aoe 21 212i 21 21 = 200 x 59 * 29 * 29 + 200 x X55 X30 + 200 x 55 20 L400* 20* (* +420 + 400) = 200 x 21 441 2 ‘| =210 = Rs. 662.03 Ans. 28. Value of the machine after 3 years 27400 (1--2)’ =77400 (1-755) 19 19 19 = 27400 X 59 X50 X20 = Rs. 23492.08 Ans. 29. Let the money borrowed by him be Rs. x ‘. Total amount of money repaid = Amount on Rs. x for 3 years + Amoonton Rs. x for 2 years + Amount on Rs. x for 1 year =x(1+ ih Ga Par (ie ~ 1007 ~~ 100, 100, oxx (2) x(n nya =EXI9 rel] xx 11x 331 3641 = F000 _ 3641 x 1000 11 x331 = Rs. 1000 Ans. 30. 4022 — 1500 = Rs. 2522 If each instalment be of Rs. x, then the principals for the amount of Rs. x for 1 year, 2 years and 3 years will be ns. (2): x (22)' ands x(22)' reece 18 Some Important Formulae : 1. In what proportion must a commodity at Rs. A per kg. be mixed with another commodity at Rs. B per kg so that the mixture may worth Rs. C perkg ? lligation . . C~B Formula—Required ratio = CHA Proof—Let x kg of commodity costing Rs. A per kg be mixed with | kg of commodity costing Rs. B per kg *. Cost of x kg of commodity at Rs. A per kg = Rs. x A and cost of 1 kg of commodity at Rs. B per kg = Rs. B Total cost of the mixture = Rs. (x A+B) and total weight of the mixture = (x+1)kg ae (+1)C = xA+B or, xC-xA = B-C B-C or, x= oD 2. In what proportion must water be mixed with milk so that on selling the mixture at cost price a profit of x% is made ? Formula—Quantity of water = Quantity of milk = x: 100 Proof—Let the quantity of milk be p litres and C.P. of milk be Rs. a per kg : CP. of plitres of milk = Rs. pxa. If q litres of water is mixed, then Volume of the mixture = p+ q litres S.P. of the mixture = Rs.(p+q)xa@ and SP.atx% = Rs. pa O22 (p+qa = a or, 100 pa + 100ga = tips pace 378A | O. Math 17. 19. 21. L A man buys milk at a certain price and after mixing it with water sells it again at the same price. How many ml of water he mixes in a litre of milk if he makes a profit of 20%? (A) 200 ml (B) 250 ml (C) 150ml (@) 20mi (E) None of these (Sub-Inspector of Police 2007) A mixture of 40 litres of milk and water contains 10% of water. How much water must be added to make the water 20% of the resulting mixture? (A) 10 litres (B) 7litres (C) Slitres (D) 3 litres (E) None of these (R.R.B. Exam. 2008) Pure ghee costs Rs. 100 per kg. After adulterating it with vegetable oil costing Rs. 50 per kg, a shopkeeper sells the mixture at Rs. 96 per kg, thereby making a profit of 20%. In what ratio does he mix the two? (A) 1:2 (B) 3:2 () 3:1 @) 2:3 (E) None of these (Hotel Management Exam. 2007) By mixing two qualities of pulses in the ratio 2 : 3 and selling the mixture at the:rate of Rs. 22 per kg, a shopkeeper makes a profit of 10%. If the cost of the smaller quantity be Rs. 14 per kg, the cost per kg of the larger quantity is— (A) Rs. 23 (B) Rs. 24 (C) Rs. 25 @) 26 (E) None of these (Hotel Management Exam. 2007) A vessel contains 14 litres of milk. From this 8 lites of milk was taken out-and replaced by water. This process was further repeated once more. How much milk is now contained in the vessel ? (A) 22 tines (B) 1S lites (C) 23 lites) 24 lites (E) None of these Answers 18) 2© 3© 4(A) 51 6© 7. 8&® 9.(C) 10.(C) 11.(A) 12.@) 13. 14.(A) 15.(C) 16. 17.(A) 18.(C) 19.(B) 20.(B) 21.) Hints C~B C-A [Here, A = 16, B = 14-50 and C = 15-40] 15.40 ~ 14.50 _ 0.90 15-40 ~ 16 0.60 332 Ans. Reqd. Ratio O. Math | 383A. Let 1 kg of each alloy are mixed and the quantity of gold in the third alloy be x kg. 7x 1.92 Lox 7 2°18 i ae . 92> 2 18 or, xe yt? ’ 9° 18° 6 5 i 1 i 2-7 -. Quantity of gold = é and quantity of copper = 6 5 = ke a 7_5 a Ratio in gold and copper =— = ¢ = 7 25 Ans. 17. Let the price of milk per litre be Re. 1 S.P. of adulterated milk per litre = Re. 1. seCeP: ” ” Quantity of water Quantity of milk DIMIND [Here, p = 1 and x = 20] 18. Quantity of water to be added # oe [Here, x= 40, y= 10 and z = 20] ©. Math | 387A 10. A rectangular garden / metres long and b metres broad is surrounded by a path x metre wide. Find the area of the path. Formula—aArea of the path = 2x{/ + b + 2x] m? Proof—AB =/ and BC = b, width of the path x a DE = /+2x and EF = b+2x Area of the path = (/+2x) (b+ 2x) — (1x b) lb + 2xl + 2xb + 4x? - lb 2x1 + 2xb + 4x? = 2x (1 +6 + 2x) sq. metres. 11. If the path is inside the garden, then area of the path = 2x (/ +b — 2x) sq. metres 12. If the length and breadth of a room are / metres and b metres respectively and two roads x metres wide each are perpendicular to each other inside the field, what is the area of the roads ? Formula— Area of both the roads = x(/+ 6—x) sq. metre " 4 a bm B A Proof— Area of the road AB = (xx Area of the road CD = bxx and area of the EFGH = xxx Area of both the roads = ix + bx—x? = x(/+ b—x) square metres. 404A | O. Math 46. 47. 48, 49. 50. The length of a rectangular plot is 75% of its breadth. If the perimeter of the plot be 1050 m, what is its area? (A) 120000 m? (B) 17500m? (C) 67500m? (D) 270000 m? (©) None of these (R.B.I. Grade (A) Exam. 2006) A room is half as broad as it is high and 1 5} times as high as it is long. If the cost of carpeting the floor at Rs. 3 per sq. m is Rs. 144, the height of the room (in meter) is— (A) 8 (B) 10 (C) 12 (D) 16 (E) None of these If the diagonal of a square be 6 inches, what in square inches will be the area of that square? (A) 9 (B) 12 (C) 18 (D) 36 (E) None of these (R.R.B. Exam. 2006) If the radii of two concentric circles are 15 cm and 13 cm respectively, the area of the annular ring in cm? is— 5 (A) 176 (B) wt (C) 88 @) 65 (E) None of these (inspectors of Income Tax & Excise etc, 2005) One side of a rectangular field is 4 metres and its length along diagonal is 5 metres. What is the area of the field? (A) 120? (B) 20m? (C) 4V 15m? (D) 15m? (E) None of these (R.R.B. Exam. 2007) Answers 1D) 20 38 4© 5@ 68) 70 8&@) 9.(A) 10.(D) 1B) 12.(A) 13.(C) 14.B) 15.) 16.(©) 17.(B) 18.(A) 19.) 20.) 21.(D) 22. 23.) 24.) 25.(C) 26.(B) 27.(A) 28.(C) 29.(B) 30.(D) 31.(C) 32.) 33.(D) 34.(C) 35.(C) 36.(C) 37.(C) 38.(D) 39.(B) 40.(D) 41.(C) 42.) 43.0 44.(A) 45.(B) 46.(C) 47.) 48.10) 49.(A) 50. (A) Hints Breadth = V 102-8? =V 100-64 = V36 = 6cm Area = 8 x6=48 cm? Ans. x= V¥225 Sm f x15 20m 20- Sm or, " and Length Length/breadth 100 22. Percentage of increase in area = tres O. Math | 409A Ans, [Here, x = 50] = (50)? = 2x50+ 100 100 + 25 125 23. Let the height of the vessel be h metres Area of iron sheets 120 = 30h or, wou > " 24, Area of the park = 2h (10 +5) +10 x5 30h + 50 120-50 70 70 30 I 23m 120 x 80 — (120-24) (80-24) = 9600 - 5376 = 4224 m? ¥3 25. Area of an equilateral triangle = 3x49 4 .= 21.22 em? 100+30) (100-30) 26. % of change in area ~ 100 Ans. a3. 2 a x (7) Ans. - 100 = +91-100=-9 = 9 (-sign shows the decrease) Ans. 27. Let the length of the big rectangle be x metres Area of the big rectangle = x x2 = 2xm? O.Math | 413A 4. Length of one side of the square = V484 = 22cm Length of wire = 4 x22 = 88cm 2nr = 88 _ 88 x7 ~ 2x2 = 14cm F 22 Area enclosed by wire = 7 14x14 = 616cm? Ans. 44. Area of 4 walls = (6 x4+5 x4) x2 = 88m" Total area of the door and window = 2.5x 1.2+1 x1 = 3+] = 4° Area of paper to be used = 88-4 = 84m? Ans. 45. Area of the floor of a square of I4ft. = 14 x14 = 196 sq.ft. saft. = 1028 Cost of | sq.ft. = 196 = Rs.55 Ans. 46. Let the breadth of the plot be xm . Length = pe eth = 709 * _ 3x = ym 1050 = 2 (+F ) x= 300m Area of the plot = 300 x2%2% xm = 67500 a“ Ans. 47. Let the height of the room be x metres Length = xx 3 2x and Breadth = Ri w O. Math | 433A WM kr Y. p.hy Mkr,P.p.hy : Wkr? pty me hy = (Jim 2. If the radius and height of a cylinder are increased by a% and b% respectively, what will be the percentage increase in its volume ? e+ 2ab _@b 100 10000 Proof—Let the initial radius and height of the cylinder be r and h respectively. and volume of second cylinder (V>) : V,:V2 Formula—% increase in volume = 2a + b+ rata, Increased radius = and increased height al 100+ a x #10046) _ | (100)? 100 _ (100 + a? x (100 + 5) ~ (100) # = wal oe 2 Increase in volume = mh (100) (1004? + 200002 + 10000b + a*b + 200ab] Teh, % increase in volume = (100) [100a? + 200004 + 10000b + ab 100 + 200ab) x Foy, @+2ab, ab 2a+ b+ 99 * T0000 Note— (1) If the radius of the base of the cylinder increases by a% but height remains same, then % increase in volume = 2a + 100 (2) If the height of the cylinder increases by b% but radius of the base remains same, then % increase in volume = b% i] 3. If the ratio of radii of the bases of two cylinders be r, : r2 and their volumes are equal, then what will be the ratio of their heights ? Formula—fy : fy = 72: 7)? Proof—Let the radii of their bases be kr, and kr, respectively and their heights be p h, and ph, respectively = V, = ner)? xphy and Vp = nr? xphy O. Math | 439A. Example 7. In a cylinder height 12cm and radius 5cm a conical hole is made, If height and radius of cone is same as that of a cylinder, then what will be total surface of rest part of cylinder? Solution : Curved surface of rest part of cylinder Curved surface of cylinder + curved surface of cone + arca of base of cylinder Wx 5x 124m x5x VE 4122 +m x5X5 120n + 65% + 25n= 2100 210 x2 = 660 cm? Ans. Example 8, Find the volume of metal in a hollow pipe 10 m long, of external-radius 4cm made of metal 1 cm thick. ( n= ) 7 Solution : Internal radius of the pipe = 4 - 1 = 3 cm Volume of metal = 2 42—-3)x 1000 cm3 = 2 x7 1000 = 22000 cm? Ans. By Short-cut Method : Volume of metal = n(rj2—ry2yh_ [Hees ri= 4,72 =4— 1=3 and h= 10 x 100cm 22 (aap — 3921 x 1000 2 (16-9) x 1000 22000 em? O. Math | 443A 22. The circumference of the base of a cone is 44 cm and the slant height is 25 cm. The volume of the cone is ( 122 + (A) 3850cm3 (B) 3696em?_(C) 1232em? D) "Sem? ©) None of these @.P.T. M.P., Exam. 2007) 23. The radii of a cylinder and a cone are equal. If the height of the cylinder is equal to the slant height of the cone, then the ratio of the curved surfaces of the cylinder and the cone is— (A) 1:1 () 2:1 (C) 3:1 (@) 4:1 (©) None of these 24, The curved surface area of one of the two cones is double of the other while the slant height of second is double that of the first. What is the ratio of the radius of the second cone to that of the first cone? (A) 1:2 @) 4:1 (C) 1:4 @) 2:1 (©) None of these (N.D.A. Exam. 2007) Answers 1(C) 2.(B) 3.(D) 4.(D) 5.(A) 6(D) 7.(B) 8.(C) 9.(D) 10.(C) 11.(A) 12.(B) 13.(A) 14.(B) 15.(B) 16.(C) 17.(C) 18.(A) 19.(C) 20.(D) 21.(B) 22.(C) 23.(B) 24.(C) Hints 1, Ratio in the perimeters of bases = 3:4 ‘ 2nr,:2t, = 3:4 noi & mn 4 & od and y= 2 Lith darth = Te 3 ar? hy ty Br hy = TP hy 3 1 “GG ) 9 32 Ans, 2. Let the radius of the base be r cm and the height be A cm r 5 Sh T= 75 or, r= he 12 12 448A | O. Math 1200 | 800_ = 40+20+ 100 * 10000 = 728 Ans. 21. Volume of the earth taken out = 1 (4)? x 14 = Bx t6xis = 704m 22 5 and area of the embankment = ° (7? -@) Height of the embankment = = 678i = 6-8 m (App.) Ans. 22. 2nr = 44 _ 44x7 T= 9xD = 7em V (25)? - (7)? = V625-49 = V576 = ae Volume of the cone = 1,2 aX Tx7x 24 = 1232cm3 Ans. 23. Let the height and radius of the cylinder be hand r respectively oe Curved surface of the cylinder = 21h and Curved surface of the cone = mt = wh(h=)) Reqd. ratio = 2:1 Ans. 24. Let radius and slant height of first cone be r, and /, while the radius and slant height of the second be r and J, ae Tel = Maly. But h= 2, is qyl, = 2nrz Xl, r= 4ry nol nm 4 Ans. 458A | O. Math 5324000.n 4000000 3 13240008, = 3 cm Per cent increase in volume of the sphere 1324000 x fe x 1000000 = 33.1% Ans. By Short-cut Method : oni 5 3a? a % increase in vol. of the sphere = 3a+—— + -— 100 * 10000 “Here, a= 10) 3x (OY , 0} = 3x 10+> 100 * 10000 304+3+0-1 = 33-1 466A | O. Math 14. Let the radius of each solid be r and the heights of the cylinder and cone be h, and hp respectively. ; 2nr +2nrh, = 3nPr r or, h= 2 or, Snr = arVhi+ 2+ nr? or, arVhy2 + 7 = Ine or, h2+P = 4° or, h? = 3P or, hy, = V3r fy fy V3r 1:2V3 Ans. 45 (2)3 (4)? = z cm Ans. 16. Vol. of the cone : Vol. of the cylinder : Vol. of the hemisphere = .1s3s2 Ans. VT, 2s (6)3 + (8)? + (R)3 = (12)3 or, R? = 1728 - 216-512 15. Rise in the water level = 5 R= 10cm Ans. 18. The volume of the cone = and volume of the sphere = 2 1 W/W) a G 1 = 3arh = or h=4r Ratio of the diameter of the sphere to the height of the cone = 2: 1 Ans. 19, Volume of the cube = a? 4 a \3 and volume of the sphere = > 2 3 en 6 O. Math | 471A 4x 120 The profitonRs.120 = BS” Rs. 3.56 Rs. 3.56 is more than Rs. 3.50 Hence it is better to invest on the second stock. Ans. Example 4. What is the market value of a 4% stock which yields 73% after paying the income tax at 5% ? Solution : Income tax on Rs. 100 = Rs. 5 Income tax on Rs. 4 = iw x4=Rs. 0.20 <. Net income after paying income tax = 4-0.20=Rs. 3.80 Rs. 7 is income on Rs. 100 Rs. 3.80 is income on Rs. = 100%3.80%5 = Rs.50 Ans. Example 5. A man invests Rs. 589500 in 3% stock at 62 3 and sells out when the Price rises to Rs. 664, He invests the proceeds in 6% stock at Rs. 106. Find the change in income. Solution : InRs. 125 ne can buy astock of Rs. 100 In Rs. 589500 he can buy a stock of = 100% 5895002 = Rs. 943200 . . 265 The increased price on Rs. 100 stock =Rs. |. ‘The increased price on Rs. 943200 = 268 x 943200 = Rs. 624870 In Rs. 106 can be bought a stock of Rs. 100 In Rs, 624870 can be bought a stock of = 100-*624870 = Rs. 589500 Income on first stock = 3x 38200 = Rs. 28296 O. Math | 493A 17. The sum of the ages of A, B and C is 96 years. To find out what is B's age, which of the following information given in the statements (a) and (b) is/are sufficient ? (a) Ais 6 years elder than C. (b) The total of B and C's age is 56 years. (A) Both (a) and (b) together are needed (B) Either (a) or (6) is sufficient (C) Only (b) is sufficient (@) Both (a) and (b) together are not sufficient (©) Only (a) is sufficient (L.LC/G.1LCJA.A.O. Exam. 2007) 18. A boat takes 6 hours to travel between two points P and Q upstream and downstream. To find out the speed of the boat in still water, which of the following information given in the statements (s), (a) and (b) is/are sufficient ? (a) The distance between the points P and Q. (b) The boat takes 4 hours to reach from P to Q upstream. (A) Only (a) is sufficient (B) Both (a) and (b) together are not sufficient (C) Both (a) and (b) together are needed (D) Only (b) is sufficient (©) Either (a) or (6) is sufficient (L.LC/G.LC/A.A.0. Exam. 2006) 19. To find out the total compound interest accrued on a sum of money after 5 years, which of the following information given in the statements (a) and (b) is/are sufficient ? (a) The rate of interest was 6 p.c.p.a. (b) The total simple interest on the same amount after 5 years at the same rate of interest will be Rs. 600. (A) Either (a) or (b) is sufficient (B) Both (a) and (b) together are needed (C) Only (a) is sufficient (D) Only (6) is sufficient ©) Both (a) and (b) together are not sufficient (L.LC/G.LC/A.A.0. Exam. 2007) 20. The area of a circle is equal to the area of a triangle. To find out the radius of the circle, which of the following information given in the statement (a) and (b) is/are sufficient 7 (a) The height of the triangle is 8 cm. (b) The hypotenus of the triangle is 10 cm. (A) Both (a) and (6) together are needed 18. 19. 21. 22. 23. }. Both the statements togetixer are not sufficient. O. Math | SOLA According to the statement (a) if the distance between the points P and Q be‘a and according to the statement (b) it takes 4 hours to travel in opposite direction of stream, then The time taken in direction of stream = 6-4 = 2 hours sag 1fa,a Speed of the boat in still water = 2 G + ‘) Hence both the statements together are needed to answer the question. From the statement (a) rate = 6% and from the statement (b) S. I. for 5 years = Rs. 600 Principal = 600 x 100 5x6 = Rs. 2000 5 3 Then C. I. for 5 years 2000 [¢ + -1] Hence both the statements together are needed to answer the question. Base of the triangle = v (hypotenuse)? — (height)? = V(10)? (8)? =6cm Area of the triangle = 5 x8x6 = 24cm? Area of the circle = 24cm? Radius of the circle = V nom Hence both the statements together are needed to answer the question. In this question the length of any of two trains is not known. Hence both the statements P and Q together are not sufficient. Let the number be 10x+ y o x+y =12 From the statement (a) x~y =4 Hence the value of x cannot be calculated From the statement (b) (10x + y) ~ (10y + x) =36 From this the value of x can not be calculated. Hence both the statements (a) and (b) together are not sufficient. Both the statements together are not sufficient. O. Math | 505A Example 4. The following graph shows the quarterly price index of a textile company during 1984-85. 360 350 340 330 §E38552 1984 1985 On the basis of the graph given above during which quarter the rate of increase of price index was the maximum ? (A) Oct. 1984 to Jan. 1985 (B) Jan. 1984 to April 1984 (C) April 1985 to July 1985 (D) Jan. 1985 to April 1985 (E) April 1984 to July 1984 Solution : % increase from Oct. 1984 to Jan. 1985 10 x 100 = "330 = 3.03 % increase from Jan. 1984 to April 1984 _4x100_ 119 ~ 335 —" % increase from April 1985 to July 1985 _12x100_ 3.45 ~ 348 77" % increase from Jan. 1985 to April 1985 8 x 100 . = 340 = 2.35 % increase from April 1984 to July 1984 _0X100_ 9 ~ 335 ~ Hence the rate of increase of price index was maximum from April 1985 to July 1985. 510A | O. Math (C) Rs. 2.5 lakhs @) Data inadequate (E) None of these 3. If the income of company B in 1988 was Rs. 3 crores, what was the total expenditure of that company in the same year ? (A) Rs. 40 lakhs (B) Rs.2crores (C) Rs. 40 crores @) Data inadequate () None of these 4. If the total expenditure of company A in 1986 and 1987 together was 7 crores, what was the total income of the same company in these two years together ? (A) Rs. 4 crores (B) Rs.2 crores (C) Rs. 1.5 crores ) Data inadequate (©) None of these Solution : 1, (B) In 1987 and 1989 because in these years the ratio is less than 1. 2. (D) : Income 3 + ® Expenditure Expenditure — 125 or, Expenditure = 735 a = Rs. 2.4 crores. 4 (D) Example 7. Directions. (1 — 5)—Study the following graph carefully and answer the questions given below it: (Bank. P.O. Exam. Bangalore 1993) Registration of New Vehicles in Delhi (in thousands) Total Vehicles zg Ss 3 8 Ss = 2 = 5 Zz . Jan. Feb. March April May = June (1991) 1. What was the increase in registration of vehicles other than Cars from January to April 1991 ? (A) 5000 (@) 10,000 = (C) 15,000 @) No increase ©) None of these O. Math | 525A Directions—(42-46) Study the table carefully and answer the questions given below it— Details of Applications Received and Candidates Selected under different Disciplines over the Years Science Selected | Recd. | Selected 42. 43. 45. In which year is the percentage of total candidates selected to total applications received the maximum ? (A) 1994 (B) 1991 (C) 1993 @) 1992 ) 1995 Under which discipline and in which year was the percentage of selected to received the lowest ? (A) Science, 1995 (B) Arts, 1992 (C) Others, 1994 @) Arts, 1993 (E) Others, 1991 Under which discipline is the percentage of total number selected with total applications received over all the years together the highest ? (A) Arts (B) Commerce (C) Science @) Other (E) Both Science and Commerce Under which discipline and in which year was the percentage increase in applications reccived over the previous year maximum ? (A) Arts, 1995 (B) Science, 1992 (C) Arts, 1994 (D) Arts, 1992 (E) None of these Under which discipline and in which year was the percentage of selected to received the highest ? (A) Science,1994 (B) Science, 1991 (C) Commerce, 1995 @) Commerce, 1992 (E) Science, 1995 Directions (47-48) Study the table given below carefully and answer each of the questions— Income tax paid by five workers in six months 540A | O. Math 110. Wi. 112. 113. 114. Sales and Earnings Report (1980-87) Sales (In lakhs of rupees) Earnings (In Rs. 10,000) Earnings Sales com co Average annual increase in sales from 1982 to 1987 (in Rs.1,00,000) (A) C1 (B) 0.2 (Cc) 03 @) 04 (E) None of these The average earnings of the store per year (in ten thousand of rupees) is most nearly equal to— (A) 85 (B) 86 (C) 87 @) 9.0 (©) 95 Percentage of earnings to sales was maximum in the year— (A) 1980 (B) 1981 (C) 1982 @) 1984 (E) None of these If the earings of 1980 is taken as base (equal to 100), what were the earnings in 1987 most nearly ? (A) 76 (B) 105 (C) 121 @) 131 (E) 145. From 1980 to 1985, in which year earnings increased by more than 100% as compared to the previous year ? (A) 1987 (B) 1986 (C) 1984 @) 1982 (E) None of these O. Math IT 5494 36. Loss due te B.S.E. Index = 840.87 -776.91 = Rs. 63.96 Increase due to Savings Bank = 1055.00 - 1027.50 = Rs. 27.50 Increase due to Dollar = 1000.32 - 999.36 = Rs.0.96 Increase due to Yen = 1196.77 — 1185.30 = Rs. 11.47 Increase due toGold = 1033.46 - 1009.62 = Rs. 23.84 Loss increase due to Silver = 1076.77 — 1047.45 = Rs. 29.32 Increase due to Pound Sterling = 1043.71 - 1032.11 = Rs. 11.60 . Maximum increase is due to Savings Bank. Ans. 37. Income on investing Rs. 2000 each on Gold, Silver and Savings Bank for 6 months = 19.24 + 153.54 + 55.00 =Rs. 227.78 and income on investing Rs. 2000 each on Dollar, Equity and Yen for | year = 0.64 — 446.18 + 393.54 Rs. 52.00 Total profit 27.78 — 52.00 =Rs, 175.78 Ans. 38. Income from Gold, Silver, Pound Sterling and Savings Bank for | year = 66.92 + 94.90 + 87.42 + 110 =Rs, 359.24 Income from Gold, Silver, Dollar and Pound Sterling = 66.92 + 94,90 + 0.64 + 87.42 =Rs. 249.88 Income from Silver, Pound Sterling, Yen and Savings Bank = 94.90 + 87.42 + 393.54 + 110.00 =Rs. 658.86 Income from Gold, Silver, Pound Sterling and Yen = 66.92 + 94.90 + 87.42 + 393.54 =Rs. 642.78 Income from Gold, Silver, Yen and Savings Bank = 66.92 + 94.90 + 393.54 + 110.00 = Rs. 665.36 Ans. 51. 52. 53. 54. 55. 56. 57. O. Math | 553A Total marks obtained by E in the given course =24+ 15+ 16+ 13+ 1248 =88 Hence the maximum marks are obtained by C. _ Ans. Angle subtended by India on the expenditure on Food = 144° and angle subtended by Pakistan on the expenditure on Food = 70° Since 144° is more than double of 70°, hence the answer (D) is correct. Ans. Amount spent by Pakistan on recreation = 120° and amount spent by Pakistan on Food = 70° Difference = 120° — 70° = 50° a; 50 x 100 Percentage of difference = 360 = 13.88 Ans. Amount spent by Indians on other items = 50° Percentage of expenditure = 50100 360 = 13.88 14 (App.) Ans, Amount spent by Indians on Journey = 45° Amonnt spent by Pakistanies on Journey = 25° Ratio =45:25 =9:5 Ans. Expenditure of Pakistan on Education = 45° and expenditure of Pakistan on Recreation = 120° 120 x 2250000 Total expenditure on Recreation = Sectorial angle made by market tax =~ ~~~ = 119° (App.) Ans. Total expenditure on rest except on defence and pension =11+15+6 and total expenditure on defence and pension = 100~32 = oe Ratio = g. =2:1(App.) Ans. O. Math | S61A Percentage increase in 1991 = o) x 100 =20 Percentage increase in 1992 = Go x 100 =0 1 ercentage increase in 1993 = a x 100 =50 Percentage increase in 1994 = negative ~. In 1990, the percentage increase is the highest. Ans, 100. Total number of students of qualified =35+50+ 20440425435 = 205 and total number of students appeared = 40+ 50+ 30+ 40+ 40440 = 240 205 x 100 ~. Reqd. percentage = 540 = 85 (App.) Ans. 101. It is clear from the graph that the percentage of qualified to appeared is lowest for IV standard. Ans, 102. There are such two pairs—I and II] where per cent is 100 & KG and IV where per cent is 85.5. Ans. 103. Reqd. percentage = a ~ = 112.37 = 110 (App.) Ans. 104. Average wage for male for 20-24 years — 29.7 +21 + 14.7 + 18 + 29.1 + 32 i 6 = 144.5. ~ 6 =Rs. 24.08 and average wage for female for 20-24 years _ 25.1 + 19.1 + 16.9 + 20.5 + 32.7 + 33.9 6 148.2 ~ 6 =Rs. 24.70 O. Math | 567A where 1; is the actual lower limit of modal class 1; is the upper limit of modal class Ff isthe frequency of modal class Si is the frequency of the previous class to the modal class and —_f is the frequency of the next class to the modal class 5. Median for ungrouped data If nis odd, then median = size of (22)m item. size of 5 th item + size of G +1 jin item If nis even then median. = ————-->—- >} — 6. ‘Median for continuous series ey Li N Median = /, + f G-) where 1, = lower limit of the median class 1, = upper limit of the median class f = frequency of the median class F = comulative frequency before the median class and N = Total of all frequencies 7. Relation between Mean, Median and Mode Mode = Mean — 3 (Mean — Median) 8. Standard Deviation for ungrouped data Where m = Arithmetic mean of 2 terms. 9. Standard Deviation for grouped data soe Py Standard deviation “ Arithmetic mean 11. Range = Largest Value of the largest variable — value of the lowest variable. 10. Coefficient of Variation = 100 Example 1. Arithmetic mean of 7, 3, 2, 4 is— (A) 2 (B) 5 (7 (@) 4 ®1 21. 24, O. Math | 575A If the mean and median of all the terms of a table be 5 and 6 respectively. What is the mode of the series ? (A)8 (B) 30 (©) 9 MO) (E) None of these Standard deviation of 1, 4, 5, 7, and 8 is 2.45. If 10 is added to each term, what is the new standard deviation ? (A) 2.45 (B) 24.5 (C) 0.245 @) 12.45 (U.P. Polytechnic, 2007) . In aclass of 43 students the height of 20 students is less than 150 cm. and the height of 16 students is more than 160 cm. The height (in cm) of the remaining seven students is as follows— 152, 154, 155, 155, 155, 158, 158 The median height of the students is— (A) 15lem (B) 154cm (C) 155cm @) 159cm (P.P.T. M.P., Exam. 2006) What is the mode of the following ? Frequenc: (A) 25.2 (B) 24.74 (C) 25.00 (@) 30.26 (E) None of these The average marks in a class of 30 students are found to be 45. On checking two mistakes were found. After correction if one student got 45 marks more and another student got 15 marks less, then the corrected average marks are— (A) 45 (B) 44 ©) 47 ©) 46 (E) None of these, (N.D.A. Exam. 2007) If the arithmetic mean of the attendance of 40 students of class A is 0.49 and the arithmetic mean of the attendance of 35 students of class B is 0.53, then the arithmetic mean of the attendances of A and B both will be— (A) 49.5% (B) 50.87% (©) 51.13% @) 51.0% ©) 49.8% The mean, mode and median of a group of 75 observations are 27, 34 and 29 respectively. Later on it was discovered that one observation was misread as 43 instead of the correct one of 63 then what are the values of the mean, mode and median after correction ? (A) 47, 34, 42.66 (B) 27.26, 44, 32.84 (C) 27.26, 54, 36.17 @) 27.26, 34, 29.51 ©) None of these O. Math | 579A . 1 a 10 A 10 Median = 3 [ Size of }thterm + Size of (+1 )thterm ] = ; [Size of Sth term + Size of 6th term] I = [16 + 18] =17 cM, ——E 1 | For median : -1,0, 1, 2, 3, 5,5, 6, 8, 10, 11 Ans. 2. Median = Size of “42+ th term = Size of 6th term =5 Since in the data 5 is repeated two times. Mode = 5 Median = Mode Ans. 2 Standard deviation = 2A _ #y 2 n =12i6_ (18 2 _ 13920 ~"46 ~\46) ~~ 529 =5.1 7+3+a+8 Ans. ay =6 584A | O. Math Ans. 26. Mean of 50 items Toial of 50 items Mean of 40 items Total of 40 items -. Total of all (40+50) items ‘ 2050 Combined mean = 90 =22.8 Ans. . we 8 27. Coefficient of variation = AM 710 =08 Ans. 28. d=10 andi = y= =3xV4 = Ans, 29. Income of each of both the towns _ 20,000 x 280 + 30,000 x 240 ~ 20,000 + 30,000 56,00,000 + 72,00,000 50,000 7 To Find The Wrong Term in A Number Series In such questions a number series is given in which the terms change from left to right in a particular order. In this series one wrong term is given i.e. this term does not change in the same order in which the other terms change. Each question is followed by a number of possible answers. The candidate has to find which of the alternative answers is in which the term is wrong in the given series. For this the candidates should study the given series carefully and find the specific order in which the terms are changing. After it, he should find which of the terms is not changing according to that specific order. Thus, the wrong term is found. Example 1. In the following number series only term is wrong. Find out the wrong term. 2, 3, 5, 8, 15, 21, 34, 55 &) 2! B) 8 (C)34 (D) 15 ®5 Solution : On studying the given series it is found that the third term is the sum of first and second term. The fourth term is the sum of second and third term. Thus, each term is the sum of its previous two terms. But in 15 this rule is not followed because the previous two terms of 15 is 5 and 8 and the sum of them is 13. Therefore, the wrong term is 15. Ans. Example 2. In the following number series, one term is wrong. After searching the wrong number, find out the correct term in its place. 95, 86, 73, 62, 47, 30, 11 (A) 65 (B) 74 (C) 47 (D) 93 (BE) 75 Solution : On studying the number series it is found that the differences of two subsequent. terms are 9, 13, 9, 15, 17 and 19. In these differences the differece between two consecutive terms is 2 except in second and third terms. If 73 is replaced by 75 then the differences between two subsequent terms of the given series will be 9, 11, 13, 15, 17 and 19 which are increasing uniformly. Hence 73 is wrong and the correct term at its place should be 75. Ans. 592A | O. Math 10. 16. Find out the number which does not fit in the following number series? 5,7, 13, 25, 44, 75, 117. (A) 7 (B) 13 (C) 25 (D) 44 (E) 75 Find out the number which does not fit in the following number series? 2, 3, 6, 15, 45, 202.5, 630 (A) 3 (B) 6 (C) 15 (D) 45 (E) 202.5 Find out the number which does nof fit in the following number series ? 696, 344, 168, 76, 36, 14, 3. (A) 344 (B) 168 (C) 76 (D) 36 (E) 14 Find out the number which does not fit in the following number series ? 778, 634, 488, 407, 358, 333, 324 (A) 634 (B) 488 (C) 407 (@) 358 (E) 333 In the following number series one term is wrong. Find out the wrong term: 1200, 1194, 1182, 1164, 1134, 1090, 1032. (A) 1194 (B) 1182 (C) 1164 @) 1134 (E) 1090 Find out the number which does not fit in the following number series ? 4, 5, 12, 38, 160, 805, 4836 (A) 5 (B) 12 (C) 38 @) 160 (E) 805 Find out the number which does not fit in the following number series ? . 2, 8,43, 210, 841, 2521, 5041 (A) 8 (B) 43 (C) 210 @) 841 () 2541 598A | O. Math 8. 12. 13. x2-1 “2-2 “2-3 «2-4 x2-5 x2-6 Hence the wrong term is 43. @ 16 24 41 66 102 151 215 +BY +P 4G ey #C7P + BP Hence the wrong term is 24. 5 7 13 25 44 75 17 L_fL_ftL__ftl_fl._tt +2 +6 +12 +20 +30 +42 L_t§ 7 tt =<‘ 1 +4 +6 +8 +10 +12 Hence the wrong term is 44. 45 202-5 630 2 3 6 15 Lf tO Zab x2 x24 x3 gh x4 Hence the wrong term is 202-5. Hence the wrong term is 76. 778 634 488 407 358 333 324 -(3)? =a)? =e ~~ =-@? = GP = GP? Hence the wrong term is 634. 1200 - 1194 = 6, 1194 — 1182 = 12, 1182 - 1164 = 18, 1164 - 1134 = 30, 1134 - 1092 = 42 and 1092 - 1032 = 60 Hence 1090 should be replaced by 1092 so that the differences of two consecutive numbers will be divisible by 6. 606A | O. Math or, x-ay =0 oi) and according to II condition x+f = b(y+n) or, x-by = bt-t «(iD From equations (i) and (ii) (b-a) y = 1-bt or, ye CADE years a-b and ce boleite 3. The age of father 1, years ago was a times the age of his son. If tz years hence the father’s age would be & times that of his son, what are their present ages ? ty (b- 1) +t, (a- 1) a-b tb) O-1 and present age of the father = a4 On), f years Proof—Let the present ages of the father and the son be x and y years respectively Formula— Present age of the son -. According tol condition x-t; = a(y-t)) or, x-ay = tat, Ai) and according to II condition x+t% = b(y+t) oS x-by © bt-t --ii) ot From equations (i) and (ii) (b-) y ty-at)—bty +h Q(b- 1+ (a-1) a-b a(t +b) (6-1) gh +, years 4. The ratio between the present ages of A and B is p: g and the sum of their present ages is a years. Find the ages of each. y years and x= xa Formula— Ageof A = 22 years 1 Bt p+q y qxa and eof B = ‘ears S p+qr™ Proof—Let the present ages of A and B be xand y years respectively «. According to I condition zs ; or, qt-py = 0 2) and according to If condition x+y = a .--(ii) . a ee. xa From equations (i) and (ii) x= pxa a p+q and ye axa O. Math | 621A 10. Let the present age of Piyush be x years and the present age of Mangesh be y years Then x = 2(y-2) and x-y = 2 8 years Ans. 1L, The present age of the son = nip—q) xt mq - np [Here,m = 6, n= 1, 1=5,p =7 and q=2]) _ L-2)x5 ~ 6X2-1x7 = 5 years Ans. iz me+tyth _ p mth th g [Here, m = 3, n=4, 1) =h=1, p=S and q=6) Bxtt+l 5 4ax+1+1 °° 6 x=l Present age of B = nxt ty Ans. 13. (Here, 1, =) =1, m=4,n=5, p=5andq=6) or 4x¢1+1 5 ’ Sx+1+1 6 2, x=2 . Presentage of Gopal = nx+t, = 5x24] = 11 years Ans. r 3x80 14, The present age of Vimal = 345 = 30 years 5x80 and present age of Aruna = 345 = 50 years sie _ 30+10 Ratio in their ages after 10 years = 50410 = 2:3 Ans. 15. Let the present age of Vishnu be x years The present age of Sunil = x +6 years UPKARES

You might also like